Download as pdf or txt
Download as pdf or txt
You are on page 1of 115

1.The Vansda National Park (VNP) is located in which state?

[A] Maharashtra
[B] Gujarat
[C] Assam
[D] Rajasthan

Hide Answer

Correct Answer: B [Gujarat ]

Notes:
The Vansda National Park (VNP) is a protected area which represents the thick woodlands of the
Dangs and southern Gujarat and is situated in the Vansda tehsil, Navsari District of Gujarat. It is
home to important animals such as Indian leopard, rhesus macaque, common palm civet, Hanuman
langur and small Indian civet. There are 443 species of flowering plants like teak, sadad, khakhro,
kadad, humb, timru, kalam, bamboo, dudhkod, mahudo, behda, umaro, kusum, tanach, asan,
shimlo, etc.

2.The Gautala Autramghat Sanctuary (GAS) is located in which state?

[A] Odisha
[B] Kerala
[C] Andhra Pradesh
[D] Maharashtra

Hide Answer

Correct Answer: D [Maharashtra ]

Notes:
The Gautala Autramghat Sanctuary (GAS) is a protected area of Aurangabad District and Jalgaon
District of Maharashtra and covers an area of 26,061.19 hectares. It houses a variety of wildlife
including chinkara, nilgai, sloth bears, jungle cat, wanderoo, leopard cat, etc.

3.Lichens, which are capable of initiating ecological succession even on a bare rock, are actually a
symbiotic association of __:

[A] algae and bacteria


[B] algae and fungi
[C] bacteria and fungi
[D] fungi and mosses

Hide Answer

Correct Answer: B [ algae and fungi]

Notes:
A lichen is a composite organism consisting of a fungus (the mycobiont) and a photosynthetic
partner (the photobiont or phycobiont) (usually algae) growing together in a symbiotic relationship.

4.Ozone layer is found in which of the following ?


[A] Troposphere
[B] Stratosphere
[C] Exosphere
[D] Thermosphere

Hide Answer

Correct Answer: B [Stratosphere]

Notes:
The ozone layer is a region of Earth’s stratosphere that absorbs most of the Sun’s ultraviolet (UV)
radiation that may cause skin cancer. It contains high concentrations of ozone (O3) in relation to
other parts of the atmosphere. Stratospheric Ozone is not harmful, but its presence on land it is
harmful.

5.In which year Project Tiger was launched in India?

[A] 1970
[B] 1973
[C] 1992
[D] 1995

Hide Answer

Correct Answer: B [1973]

Notes:
Project Tiger is a tiger conservation programme launched in April 1973 by the Government of India
during Prime Minister Indira Gandhi’s tenure. Please note Project Elephant was launched in 1992

6.Which of the following is not among the four coral reef regions of India identified by the
Government for intensive conservation and management?

[A] Gulf of Mannar


[B] Gulf of Khambat
[C] Lakshadweep Islands
[D] Andaman and Nicobar Islands

Hide Answer

Correct Answer: B [Gulf of Khambat]

Notes:
The four coral reef areas identified for intensive conservation and management in the country are
Gulf of Mannar, Gulf of Kachchh, Lakshadweep and Andaman and Nicobar Islands.

7.Which of the following factors are responsible for coral bleaching?

1. Increased or reduced water temperature

2. Over fishing

3. Increased solar irradiance

4. Changes in salinity

Select the correct option from the codes given below:


[A] Only 1, 2 & 4
[B] Only 1, 3 & 4
[C] Only 2, 3 & 4
[D] 1, 2, 3 & 4

Hide Answer

Correct Answer: D [ 1, 2, 3 & 4]

Notes:
All the above factors are responsible for coral beaching. Elevated sea temperatures are the biggest
culprits when it comes to coral bleaching. El Niño also plays a role in heating up ocean waters. Over
fishing results in increase in zooplanktons which results in low availability of oxygen leads to death of
corals. Extra-bright sunlight, especially when combined with the aforementioned extra-warm
seawater another important cause of coral bleaching.

8.Which of the following is / are results of the excessive use of nitrogenous fertilizers?

1. Increased acidity of soil

2. Leaching of nitrate to the ground-water

3. Depletion of Stratospheric Ozone

Select the correct option using the code given below:


[A] Only 1 & 2
[B] Only 2 & 3
[C] Only 1 & 3
[D] 1, 2 & 3

Hide Answer

Correct Answer: D [ 1, 2 & 3]

Notes:
Excessive use of fertilizers containing Nitrogen in the form of ammonia or other forms subject to
nitrification would produce soil acidity. Second statement is also correct, the excessive nitrates
would leach to groundwater and once nitrates have leached to groundwater, it is very difficult to
remove it from this water. Third statement is also correct because N2O is an ozone depletation
substance. Kindly note that N2O also forms Ground level ozone due to its reaction to volatile organic
compounds (VOCs).

9.Which of the following is the correct definition of the Primary Productivity?

[A] Rate of photosynthesis in plants and photosynthetic bacteria as compared to the rate of plant
consumption by herbivores
[B] Total amount of light energy converted to organic compounds in a given area per unit time.
[C] Rate of decomposition by the detritivores.
[D] Total biomass of the photosynthetic organisms in the ecosystem.

Hide Answer
Correct Answer: B [ Total amount of light energy converted to organic compounds in a given area
per unit time.]
Notes:
B is the correct answer.

10.The World Bank Green Bonds:

1. are credit fixed income product

2. allow both public and private sectors to invest

3. are denominated only in reserve currencies

Which of the above statements is/are correct?

[A] Only 1 & 2


[B] Only 2 & 3
[C] Only 1 & 3
[D] 1, 2 & 3

Hide Answer

Correct Answer: A [ Only 1 & 2 ]

Notes:
Third statement is incorrect because the World Bank Green Bonds are denominated in 18 currencies
at present.

11.A large part of global greenhouse gas emissions are attributed to the methane gas production in
farming and livestock management. Which among the following practices in agriculture lead to
methane emission?
1. Composting of organic waste
2. Open burning of crop residues on fields
3. Addition of Urea and Lime in soil
4. Rice / Paddy cultivation
Select the correct option from the codes given below:

[A] Only 1, 2 & 3


[B] Only 2, 3 & 4
[C] Only 1, 2 & 4
[D] 1, 2, 3 & 4

Hide Answer

Correct Answer: C [ Only 1, 2 & 4]


Notes:

12.Which of the following is/ are declared by state governments?

1. National Parks

2. Wildlife Sanctuaries

3. Tiger Reserves

Select the correct option from the codes given below:

[A] Only 2
[B] Only 1 & 2
[C] Only 2 & 3
[D] 1, 2 & 3

Hide Answer

Correct Answer: B [ Only 1 & 2]

Notes:
Please note that Tiger Reserves come under the “Project Tiger” that was launched in April, 1973. It
was later given statutory backing by amendment of the Wildlife Protection Act in 2006 and thus
National Tiger Conservation Authority was created. The NTCA is responsible for implementation of
the Project Tiger plan to protect endangered tigers. It comes under Central Government, so central
government declares the tiger reserves, though state governments can send their proposals and
requests to NTCA.

13.Consider the following differences between habitat and niche:

1. Habitat is larger in size in comparison to niche

2. Habitat supports number of species, niche supports single species

Which of the above statements is/are correct?


[A] 1 Only
[B] 2 Only
[C] Both 1 & 2
[D] Neither 1 nor 2

Hide Answer

Correct Answer: C [ Both 1 & 2]

Notes:
Both are correct statements

14.Consider the following species of bears:

1. Brown bear

2. Asiatic black bear

3. Sloth Bear

Which of the above can be spotted or seen in India in natural habitats?

[A] 1 & 2 Only


[B] 2 Only
[C] 2 & 3 Only
[D] 1, 2 & 3

Hide Answer

Correct Answer: D [ 1, 2 & 3]

Notes:
Asiatic black bears live in forested mountain habitats (1,200-3300 metres) in the states of Jammu
and Kashmir, Himachal Pradesh, Uttaranchal, Arunachal Pradesh, Sikkim, West Bengal, Mizoram,
Meghalaya, and Tripura. Asian black bears are listed as vulnerable by IUCN. Brown bear species is
distributed across much of northern Eurasia and North America. Brown bear remains listed as a least
concern species by the IUCN. Recently, eight brown bears were sighted including three adult females
and five grown-up cubs, is a rare record in kargil area of Jammu & Kashmir. Sloth bear is a nocturnal
species native to Indian subcontinent. Sloth bears have been listed as vulnerable by IUCN.

15. Consider the following statements about the National Adaptation Fund for Climate Change
(NAFCC):

1. The Central Pollution Control Board has been appointed as the National Implementing Entity
(NIE) of the fund

2. The Fund is used to assist State and Union Territories that are particularly vulnerable to the
adverse effects of climate change

Which of the above statements is/are correct?

[A] 1 Only
[B] 2 Only
[C] Both 1 & 2
[D] Neither 1 nor 2
Hide Answer

Correct Answer: B [ 2 Only]

Notes:
First statement is incorrect because the National Bank for Agriculture and Rural Development
(NABARD) has been appointed as National Implementing Entity (NIE) responsible for implementation
of adaptation projects under the (NAFCC). Second statement is correct because the fund is used to
assist State and Union Territories that are particularly vulnerable to the adverse effects of the
climate change.

16.Which of the following is/are the species of orangutans?

[A] Bornean
[B] Sumatran
[C] Tapanuli
[D] All of the above

Hide Answer

Correct Answer: D [All of the above]

Notes:
India’s only orangutan, ‘Binny’, has passed away at the age of 41 at Nandankanan Zoological Park in
Odisha. The great ape was brought to the Odisha zoo in November 2003 from Pune’s Rajiv Gandhi
Zoological Park on November 20, 2003, when she was 25. Orangutans are one of the world’s three
extant species of great apes. According to the World Wide Fund for Nature (WWF), there are three
species of orangutans — Bornean, Sumatran and Tapanuli. Bornean and Sumatran orangutans differ
a little in appearance and behaviour. All three kinds of orangutans are listed as “Critically
Endangered” by the United Nations-affiliated International Union for Conservation of Nature (IUCN).
They are native to Indonesia and Malaysia and at present are found only in the rainforests of Borneo
and Sumatra. The WWF states that orangutans, with distinctive red fur, are the largest arboreal
mammal, spending most of their time in trees.They are considered to be among the most intelligent
primates. They can live up to 45 years in the wild and even more in zoos.

17.What will be the effect of the destruction of the phytoplankton in the sea?

1. The number of zooplanktons will increase

2. The amount of oxygen will decrease

3. Increase in the number of primary consumers

Select the correct answer from the codes given below-

[A] 1,2,3
[B] 2 only
[C] 1 and 2 only
[D] 2 and 3 only

Hide Answer

Correct Answer: B [2 only]


Notes:
Phytoplanktons are autotrophic organism and is the main food of zooplanktons. They make their
food by using carbon dioxide and release oxygen. So their destruction will decrease the number of
zooplanktons in the sea and will decrease the oxygen in the water.

18.Chirang Ripu Elephant Conservation reserve is found in whcih of the following states?

[A] Assam
[B] Arunachal Pradesh
[C] West Bengal
[D] Madhya Pradesh

Hide Answer

Correct Answer: A [Assam]

Notes:
It is one of the sites under MIKE program and is in Assam.

19.Which of the following is true about Petcoke?

1. It is a byproduct of petroleum refining

2. It is responsible releasing high sulphur content

Select the correct answer from the codes given below

[A] 1 only
[B] 2 only
[C] Both
[D] None

Hide Answer

Correct Answer: C [Both]

Notes:
Petcoke is the bottom of the barral by-product of petroleum refining. It is cheap and easier to
transport, having high calorific value. They are used as a fuel. It is a pollutant as it release high
sulpher content and fine dust.

20.The organisms which occur primarily or most abundantly in the ecotone are referred to as?

[A] Edge population


[B] Edge species
[C] Edge community
[D] None of the above

Hide Answer

Correct Answer: B [Edge species]

Notes:
The organisms which occur primarily or most abundantly in the ecotone are referred to as edge
species. In the terrestrial ecosystems edge effect is especially applicable to the birds. 21.Which
among the following statements are applicable to both Nepenthes khasiana & Tropical sundew
(Drosera burmannii) plants?
1. Both are endangered plant species
2. Both are found in Eastern Himalaya
3. Both are insectivorous plant species
Select the correct option from the codes given below:

[A] Only 1 & 2


[B] Only 2 & 3
[C] Only 1 & 3
[D] 1, 2 & 3

Hide Answer

Correct Answer: D [1, 2 & 3]

Notes:
Kindly note that both these plants are found in Khasi hills of north east India.

22.Which of the following countries has signed a ‘New Delhi Declaration on Asian Rhinos 2019?
[A] Malaysia
[B] Bhutan
[C] Indonesia
[D] All of the above

Hide Answer

Correct Answer: D [All of the above]

Notes:
‘The New Delhi Declaration on Asian Rhinos 2019’ has recently signed by the five rhino range nations
– India, Bhutan, Nepal, Indonesia and Malaysia – for the conservation and protection of the species
at the recently held 2nd Asian Rhino Range Countries meeting in New Delhi. The declaration was
signed to conserve and review the population of the Greater one-horned, Javan and Sumatran
rhinos every four years to reassess the need for joint actions to secure their future. The declaration
includes undertaking studies on health issues of the rhinos, their potential diseases and taking
necessary steps; collaborating and strengthening wildlife forensics for the purpose of investigation
and strengthening of transboundary collaboration among India, Nepal and Bhutan for the
conservation and protection of the Greater one-horned rhino. The meet was organized by the
Ministry of Environment, Forest and Climate Change (MoEFCC) in collaboration with IUCN Asian
Rhino Specialist Group, WWF- India and Aaranyak.
23.A site containing 220-million-year-old fossilized remains of dinosaurs has recently discovered in
which country?

[A] Iran
[B] Argentina
[C] New Zealand
[D] Australia

Hide Answer

Correct Answer: B [Argentina]


Notes:
A site containing the 220-million-year-old fossilized remains of nearly a dozen dinosaurs has been
discovered in western Argentina. According to Argentinian paleontologist Ricardo Martinez of the
University of San Juan, there are almost ten different individuals, with practically no sediment. The
discovery is doubly important because there are at least 7 or 8 individuals of dicynodonts, the
ancestors of mammals, the size of an ox. There were also remains of archosaurs, reptiles that could
be the ancestors of great crocodiles. The find was discovered in September 2018 in San Juan
province, about 1,100 kilometres west of Buenos Aires. The site is between one and two metres in
diameter and about the same depth, leading scientists to speculate it was a former drinking hole at a
time of great drought, and the creatures died of weakness at the spot. Argentina has been a rich
source of fossils from the Triassic, Jurassic and Cretaceous eras over the years – of most creatures
not found in the northern hemisphere.

24.Which of the following is not correct about Monosodium glutamate? 1.It is an amino acid found
in your body and most foods. 2.It is a flavour enhancer. 3.It causes a condition called Chinese
restaurant syndrome.
[A] 1 only
[B] 1 and 2 only
[C] 1,2 and 3
[D] None of the above

Hide Answer

Correct Answer: D [None of the above]

Notes:
MSG is a flavour enhancer commonly added to Chinese food, canned vegetables, soups, and
processed meats. It is claimed that MSG can cause asthma and even brain damage.

25.Animal Welfare Board was established in which year?

[A] 1962
[B] 1963
[C] 1964
[D] 1965

Hide Answer

Correct Answer: A [1962]

Notes:
The Animal Welfare Board of India was established in the year 1962 under Section 4 of the
Prevention of Cruelty to Animals Act,1960. Rukmini Devi Arundale as its first chairman.

26.Which of the following can be classified under Primary pollutants?

1. Plastic

2. Sulphur Dioxide

3. Nitrogen oxide

4. Ozone
Select the correct answer from the codes given below-

[A] 1,2,3,4
[B] 1,2 and 3 only
[C] 2,3 and 4 only
[D] 1 and 4 only

Hide Answer

Correct Answer: B [
1,2 and 3 only]

Notes:
Primary pollutants are those pollutants that effect the environment in their unmodified form.
Example- Carbon monoxide, sulphur dioxide, nitrogen oxide etc.
Secondary pollutants are those pollutants that effect the environment in their modified forms.
Example- Troposphere ozone is formed after the reaction of Volatile Organic Compound and
nitrogen oxides.

27.Which of the following is true about the Persistant Organic Pollutants?

1. They remain intact for a long period of time

2. They are capable of bio-accumulation

Select the correct answer from the codes given below-

[A] 1 only
[B] 2 only
[C] Both
[D] None

Hide Answer

Correct Answer: C [Both]

Notes:
Persistant Organic Pollutants are harmful organic chemical substances that remain intact for a very
long period of time and becomes widely distributed over a large area. They are capable of
accumulation and is found in the higher trophic level. They are toxic to both human and wildlife.

28.Which of the following statements are correct about Desert Biome?


1) low and sporadic rainfall with low humidity.
2) The days are very hot but the nights are cold.
3) Euphorbias and Sagebrush are found.
Select the correct answer from the codes given below:

[A] Only 1 & 2


[B] Only 2
[C] Only 2 & 3
[D] 1, 2 & 3

Hide Answer

Correct Answer: C [Only 2 & 3]


Notes:
Desert Biomes are present in Continental interiors with very low and sporadic rainfall with low
humidity. The days are very hot but the nights are cold. The flora is drought resistance such as
cactus, euphorbias, sagebrush.

29.Which of the following statement are correct regarding the Savanna Ecosystem?
1) The rainfall is about 500 mm per year.
2) The dry season lasts for six to eight months.
Select the correct option from the codes given below:

[A] Only 1
[B] Only 2
[C] Both 1 & 2
[D] Neither 1 & 2

Hide Answer

Correct Answer: C [Both 1 & 2]

Notes:
Dry savanna grazing ecosystems have developed in the central and eastern parts of Rajasthan,
where the rainfall is about 500 mm per year and the dry season is of six to eight months.

30.Which of the following wetlands of India are in Montreux record?


1) Chilika lake
2)Keoladeo National Park
3) Loktak Lake
Select the correct option from the codes given below:

[A] Only 1
[B] Only 1 & 2
[C] Only 2 & 3
[D] Only 3

Hide Answer

Correct Answer: C [Only 2 & 3]

Notes:
Currently, there are two wetlands of India which are present in the Montreux record:
1) Keoladeo National Park, which is located in Rajasthan
2) Loktak Lake, Which is located in Manipur

31.Which of the following are the environmental benefits of estuarine ecosystem?


1. Estuaries help in water quality regulation and groundwater recharge.
2. They are the habitat, breeding and nursery grounds for plants and animals.
3. They are useful for commercial fishing.
Select the correct option from the codes given below:

[A] Only 1 & 2


[B] Only 2 & 3
[C] Only 1 & 3
[D] 1, 2 & 3
Hide Answer

Correct Answer: D [1, 2 & 3]

Notes:
Estuaries help in water quality regulation and groundwater recharge. They are the habitat, breeding
and nursery grounds for plants and animals. They are useful for commercial fishing.

32.Which of the following statements are correct regarding mangroves?


1. Mangroves require low solar radiation.
2. They have the ability to absorb fresh water from saline/brackish water.
3. Mangroves produces pneumatophores (blind roots) to overcome respiration problem in the
anaerobic soil conditions.
Select the correct option from the codes given below:

[A] Only 1 & 2


[B] Only 2 & 3
[C] Only 1 & 3
[D] 1, 2 & 3

Hide Answer

Correct Answer: B [Only 2 & 3]

Notes:
Mangroves require high solar radiation. They have the ability to absorb fresh water from
saline/brackish water. Mangroves produces pneumatophores (blind roots) to overcome respiration
problem in the anaerobic soil conditions.

33.In Gujarat mangroves Avicennia marine, Avicennia officinalis and Rhizophora mucronata are
found mainly found in which of the following regions?
1. Gulf of Kachchh
2. Kori creek
Select the correct option from the codes given below:

[A] Only 1
[B] Only 2
[C] Both 1 & 2
[D] Neither 1 & 2

Hide Answer

Correct Answer: C [Both 1 & 2]

Notes:
In Gujarat mangroves Avicennia marine, Avicennia officinalis and Rhizophora mucronata are found
mainly found in the following regions:
1. Gulf of Kachchh
2. Kori creek

34.Which of the following statements are correct regarding mangroves in India?


1. Mangroves occur along the intertidal region of estuaries and creeks in Maharashtra, Goa and
Karnataka.
2. Small tidal estuaries, neritic inlets and the lagoons support a dense and diverse undisturbed
mangrove flora on Andaman & Nicobar Islands.
Select the correct option from the codes given below:

[A] Only 1
[B] Only 2
[C] Both 1 & 2
[D] Neither 1 & 2

Hide Answer

Correct Answer: C [Both 1 & 2]

Notes:
Mangroves occur along the intertidal region of estuaries and creeks in Maharashtra, Goa and
Karnataka. Small tidal estuaries, neritic inlets and the lagoons support a dense and diverse
undisturbed mangrove flora on Andaman & Nicobar Islands.

35.Which of the following are the roles played by mangroves?


1. Mangroves protects coastal lands from tsunami, hurricanes and floods.
2. Mangroves help in moderating monsoonal tidal floods and reduce inundation of coastal lowlands.
3. Mangrove do not support much flora, avifauna and wild life.
Select the correct option from the codes given below:

[A] Only 1 & 2


[B] Only 2 & 3
[C] Only 1 & 3
[D] 1, 2 & 3

Hide Answer

Correct Answer: A [Only 1 & 2]

Notes:
Mangroves protects coastal lands from tsunami, hurricanes and floods. Mangroves help in
moderating monsoonal tidal floods and reduce inundation of coastal lowlands. Mangrove support
rich flora, avifauna and wild life.

36.Which of the following lead to the destruction of mangroves?


1. Conversion of area for agricultural
2. Mining
3. Oil spills
4. Aquaculture
Select the correct option from the codes given below:

[A] Only 1, 2 & 3


[B] Only 2, 3 & 4
[C] Only 1 ,2 & 4
[D] 1, 2, 3 & 4

Hide Answer

Correct Answer: 0 []
Notes:
Mangroves are destroyed because of a number of factors such as conversion of area for agricultural
purpose, fuel, fodder and, salinization, minig, oil spills, aquacultural (shrimp farming), use of
chemical pesticides & fertilizers, industrial purposes.

37.Which of the following statements are correct regarding Bluefin Tuna?


1. Its IUCN status endangered.
2. It is found in the Coral Triangle.
3. There are three species of bluefin tuna — Atlantic, Pacific and Southern.
Select the correct option from the codes given below:

[A] Only 1 & 2


[B] Only 2 & 3
[C] Only 1 & 3
[D] 1, 2 & 3

Hide Answer

Correct Answer: D [1, 2 & 3]


Notes:
Bluefin Tuna’s IUCN status endangered and is found in the Coral Triangle. There are three species of
bluefin tuna — Atlantic, Pacific and Southern. It weighs around 1500 pounds.

38.What is the IUCN status of Nawhal?

[A] Threatened
[B] Near Threatened
[C] Endangered
[D] Critically Endangered

Hide Answer

Correct Answer: B [Near Threatened]

Notes:
The IUCN status of Nawhal is Near Threatened. They have a unicorn-like tusk. They spend most of
their time in the Arctic waters of Canada, Greenland, Norway and Russia. The tusk of Nawhal is
actually a tooth and can grow up to 12 feet (3.7 meters).

39.Which of the following statements are correct about Ploughshare tortoise?


1. It is a critically endangered species of tortoise.
2. It is endemic to Madagascar.
3. Its scientific name is Astrochelys yniphora ans is also known as the angonoka, ploughshare
tortoise, Madagascar tortoise, or Madagascar angulated tortoise.
Select the correct option from the codes given below:
[A] Only 1 & 2
[B] Only 2 & 3
[C] Only 1 & 3
[D] 1, 2 & 3

Hide Answer
Correct Answer: D [1, 2 & 3]

Notes:
Ploughshare tortoise is a critically endangered species of tortoise. It is endemic to Madagascar. Its
scientific name is Astrochelys yniphora ans is also known as the angonoka, ploughshare tortoise,
Madagascar tortoise, or Madagascar angulated tortoise.

40.What is the IUCN status of the La Hotte glanded frog?

[A] Endangered
[B] Extinct
[C] Critically Endangered
[D] Least Concern

Hide Answer

Correct Answer: C [Critically Endangered]

Notes:
The scientific name of the La Hotte glanded frog is Eleutherodactylus glanduliferis. It is a species of
frog which belongs to the family Eleutherodactylidae. the IUCN status of the La Hotte glanded frog is
Critically Endangered.
41.Which of the following are the objectives of the Global Tiger Forum?
1) To promote a worldwide campaign to save only the tiger and not its prey.
2) To promote a legal framework in the countries involved in biodiversity conservation.
3) Training and Research promotion.
Select the correct answer from the codes given below:

[A] Only 1
[B] Only 2 & 3
[C] Only 1 & 3
[D] Only 3

Hide Answer

Correct Answer: B [Only 2 & 3]

Notes:
Some of the objectives of the Global Tiger Forum are:
1) To promote a worldwide campaign to save the tiger, its prey and its habitat
2) Training and Research promotion.
3) To promote a legal framework for the countries which are involved in biodiversity conservation.

42.How many Chemicals are listed in Annex III of the Rotterdam Convention?

[A] 12
[B] 28
[C] 43
[D] 54

Hide Answer

Correct Answer: C [43]


Notes:
There are a total of 43 chemicals listed in Annex III of the Rotterdam convention. It includes
pesticides and industrial chemicals that have been banned or severely restricted for health or
environmental reasons by two or more Parties.

43.The Vienna Convention for the protection of the Ozone Layer was adopted in which of the
following year?

[A] 1944
[B] 1985
[C] 1995
[D] 2005

Hide Answer

Correct Answer: B [1985]

Notes:
The Vienna Convention for the Protection of the Ozone Layer is a multilateral environmental
agreement. It was adopted in the year 1985 and entered into force in 1988. It does not include
legally binding reduction goals.

44.Deo Bhumi and Bugyal are the local names of sacred groves of which of the following states?

[A] Jammu & Kashmir


[B] Uttar Pradesh
[C] Uttarakhand
[D] BIhar

Hide Answer

Correct Answer: C [Uttarakhand]

Notes:
Deo Bhumi and Bugyal are the local names of sacred groves of Uttrakhand. They are total 22 in
number. They are sacred alpine meadows. These groves are dedicated to local deities of Uttrakhand
such as Chandrabadni Devi, Hariyali Devi, Kotgadi Ki Kokila Mata, Pravasi Pavasu Devata, Devrada
and Saimyar and are believed to be protected by them.

45.Which of the following gas is emitted naturally by the soil?

[A] Carbon Dioxide


[B] Radon
[C] Ozone
[D] Formaldehyde

Hide Answer

Correct Answer: B [Radon]


Notes:
Radon is a gas that is emitted naturally by the soil and because of modern houses which have poor
ventilation, it is confined inside the house and causes lung cancers. Its symbol is Rn and atomic
number 86.
46.Which of the following statements are correct regarding the Wildlife Crime Control Bureau?
1) It is a constitutional body.
2) It was established in 2007.
Select the correct option from the codes given below:

[A] Only 1
[B] Only 2
[C] Both 1 & 2
[D] Neither 1 & 2

Hide Answer

Correct Answer: C [Both 1 & 2]

Notes:
Wildlife Crime Control Bureau is a statutory multi-disciplinary body under the Union Ministry of
Environment, Forests and Climate Change (MoEFCC). It was established in 2007 to combat organized
wildlife crime in the country.

47.Which of the following are Environmentally Sensitive Places?


1. Religious and historic places
2. Archaeological monuments/sites
3. Biosphere reserves
4. Airport
Select the correct option from the codes given below:

[A] Only 1, 2 & 3


[B] Only 2, 3 & 4
[C] Only 1, 3 & 4
[D] 1, 2, 3 & 4

Hide Answer

Correct Answer: D [1, 2, 3 & 4]

Notes:
Some of the Environmentally Sensitive Places are as follows:
1. Religious and historic places
2. Archaeological monuments/sites
3. Biosphere reserves
4. Airport
5. Natural lakes, swamps Seismic zones tribal Settlements
6. Hill resorts/mountains/ hills

48.How many wildlife sanctuaries are there in the state of Bengal?

[A] 12
[B] 15
[C] 16
[D] 19

Hide Answer

Correct Answer: B [15]


Notes:
There is 6 national parks and 15 wildlife
sanctuaries in the state which are constituted under Wildlife Protection
Act, 1972.

49.The toxic metal associated with the Minamata episode is ____:

[A] Lead
[B] Arsenic
[C] Mercury
[D] Cadmium

Hide Answer

Correct Answer: C [Mercury]

Notes:
Minamata disease is a neurological syndrome caused by severe methyl mercury poisoning. It is
caused by eating large quantities of fish and shellfish polluted by methyl mercury in factory
wastewater. The disease is named after Minamata city in Kumamoto prefecture, Japan, where it was
first discovered in 1956.

50.Green manure is obtained from _____:

[A] Oil seed husk cakes


[B] Decomposing green legume plants
[C] Domestic vegetable waste
[D] Fresh animal excreta

Hide Answer

Correct Answer: B [Decomposing green legume plants]

Notes:
Green undecomposed material used as manure is called green manure. It is obtained in two ways:
(a) growing green manure crops usually belonging to leguminous family and incorporating into the
soil after sufficient growth or (b) by collecting green leaf from plants grown in wastelands, field
bunds and forest. The most important green manure crops are sunnhemp, dhaincha, etc.

1.Which among the following two gases contribute to begin the formation of stars?

[A] Hydrogen & Nitrogen


[B] Hydrogen & Helium
[C] Hydrogen & Oxygen
[D] Nitrogen & Helium

Hide Answer

Correct Answer: B [Hydrogen & Helium]


Notes:
The formation of the stars is explained by Big bang theory. The big bang produced hydrogen and
helium. Most of the heavier elements are created only by the thermonuclear fusion reactions in
stars, so they would not have been present before the first stars had formed.

2.World’s oldest co-educational boarding school Dollar Academy is located in which of the following
countries?

[A] United States


[B] Scotland
[C] Switzerland
[D] France

Hide Answer

Correct Answer: B [Scotland]

Notes:
World’s oldest co-educational boarding school Dollar Academy is located in Scotland. It is founded in
1818 by John McNabb, is an independent co-educational day and boarding school in Scotland.

3.Which among the following is the most recent in the geological time scale of the International
Commission on Stratigraphy?

[A] Tertiary Period


[B] Quaternary Period
[C] Cretaceous Era
[D] Oligocene

Hide Answer

Correct Answer: B [Quaternary Period]

Notes:
Quaternary is the current and most recent of the three periods of the Cenozoic Era in the geologic
time scale. It follows the Neogene Period and spans from 2.588 ± 0.005 million years ago to the
present.

4.A star at which among the following locations would appear to twinkle more than others
normally?

[A] Straight up in Observer’s line of sight


[B] Stars closer to the horizon
[C] Stars closer to poles
[D] None of them

Hide Answer

Correct Answer: B [Stars closer to the horizon]


Notes:
Stars seem to twinkle or change their brightness all the time. In fact, most stars shine with a steady
light. The movement of air (sometimes called turbulence) in the Earth`s atmosphere causes the
starlight to get slightly bent as it travels from the distant star through the atmosphere to us on the
ground. Some of the light reaches us directly but some gets bent slightly. This gives the illusion of
twinkling. Stars closer to the horizon appear to twinkle more than others. This is because the
atmosphere is a lot denser near the horizon than between the Earth and a star higher in the sky.

5.The Amazon Canyon is located in which ocean?

[A] Pacific Ocean


[B] Arctic Ocean
[C] Atlantic Ocean
[D] Indian Ocean

Hide Answer

Correct Answer: C [Atlantic Ocean]

Notes:
The Amazon Canyon is located in the Atlantic Ocean. It is a submarine canyon within the Amazon
Fan in the Atlantic Ocean, nearly 200 miles from the mouth of the Amazon River, near South
America.

6.Which is the deepest river in the world?

[A] Loire River


[B] Nile River
[C] Rhine River
[D] Congo River

Hide Answer

Correct Answer: D [Congo River]

Notes:
The Congo River is the deepest river in the world. It was formerly known as the Zaire River. It is also
the second longest river in Africa and world’s second largest river in terms of discharge volume of
water.

7.Which ancient astronomer made the first attempt to create a heliocentric model of universe?

[A] Aristarchus of Samos


[B] Claudius Ptolemaeus
[C] Aristotle
[D] Eudoxus of Cnidus

Hide Answer

Correct Answer: A [Aristarchus of Samos]

Notes:
Aristarchus of Samos, a Greek astronomer made the first attempt to create a heliocentric model of
universe. He places the sun at the centre, while the Earth revolved around the Sun in a circular path.

8.What of the following device is used to measure the diameter of stars?

[A] Barometer
[B] Interferometer
[C] Photometer
[D] Viscometer

Hide Answer

Correct Answer: B [Interferometer]

Notes:
Interferometer is used in astronomy to measure the diameters of the stars. It was invented in the
late 19th century by Albert Michelson

9.The Betelgeuse star is located in which constellation?

[A] Sirius
[B] Orion
[C] Lyra
[D] Ursa Major

Hide Answer

Correct Answer: B [Orion ]

Notes:
The Betelgeuse star is located in the constellation Orion. It is a Supergiant star. Its diameter is larger
than the diameter of Jupiter’s orbit around the Sun.

10.Which period is not a part of the Paleozoic Era?

[A] Devonian
[B] Triassic
[C] Cambrian
[D] Permian

Hide Answer

Correct Answer: B [Triassic]


Notes:
The Triassic period is a part of the Mesozoic Era. The Paleozoic Era is divided into six periods –
Permian, Carboniferous, Devonian, Silurian, Ordovician and Cambrian

11.Gulf of Thailand is the part of which sea?

[A] Red Sea


[B] South China Sea
[C] Mediterranean Sea
[D] Caspian Sea

Hide Answer

Correct Answer: B [South China Sea]

Notes:
Gulf of Thailand is the part of the South China Sea. It is also known as the Gulf of Siam. It is a
marginal water body in the western Pacific Ocean. It is surrounded by Cambodia and Vietnam in the
northeast, and by Thailand in the rest part.
12.Which is the second longest canyon in the world?

[A] Echo Canyon


[B] Amazon Canyon
[C] Ulubey Canyon
[D] Fish River Canyon

Hide Answer

Correct Answer: C [Ulubey Canyon]

Notes:
Ulubey Canyon is the second longest canyon in the world. It is located in the Ulubey district of Usak
Province in Turkey. It covers an area of 119 hectares.

13.What is the diameter of the Milky Way?

[A] 30,000 light years


[B] 10,000 light years
[C] 1,20,000 light years
[D] 20,00,000 light years

Hide Answer

Correct Answer: C [1,20,000 light years ]


Notes:
The Milky Way is a barred spiral galaxy with a diameter of 1,20,000 light years. It is estimated to
contain 100-400 billion planets.

14.Which among the following Galaxies is also known as Messier 31?

[A] Milky Way galaxy


[B] Pinwheel galaxy
[C] Andromeda galaxy
[D] Sombrebo galaxy

Hide Answer

Correct Answer: C [Andromeda galaxy]

Notes:
Andromeda Galaxy is also known as Messier 31. It is a barred spiral galaxy. Andromeda is the nearest
major galaxy to the Milky Way.

15.Which among the following are gaseous planets?

[A] Mercury, Venus, Earth, Mars


[B] Mercury, Jupiter, Saturn, Uranus
[C] Saturn, Uranus, Neptune, Pluto
[D] Jupiter, Saturn, Uranus, Neptune

Hide Answer

Correct Answer: D [Jupiter, Saturn, Uranus, Neptune]


Notes:
Gaseous planets also called as outer planets or Jovian planets include Jupiter, Saturn, Uranus and
Neptune. They are made up of hot gases mainly hydrogen and helium.

16.Which is the lowermost and the oldest epoch of the tertiary period of the geologic time scale?

[A] Eocene
[B] Holocene
[C] Cretaceous
[D] Palaeocene

Hide Answer

Correct Answer: D [Palaeocene]

Notes:
Palaeocene is the lowermost and oldest epoch of the tertiary period. It spans between 65 million
years ago to 57 million years ago.

17.Which among the following is an example of Transform Plate Boundary?

[A] San Andreas Fault, California


[B] Andes, South America
[C] Mount Fuji, Japan
[D] Himalayas, India

Hide Answer

Correct Answer: A [San Andreas Fault, California]

Notes:
The plates slide past each other at the transform plate boundary. Here the crust is neither created
nor destroyed. E.g. San Andreas Fault.

18.Which among the following landforms resemble those formed by solution in some karst areas of
limestone?
[A] Pingo
[B] Nivation Hollows
[C] Thermokarst
[D] Hummocks

Hide Answer

Correct Answer: C [Thermokarst]

Notes:
Thermokarst are formed of small hummocks and marshy hollows which resemble the landforms
formed by solution in the karst areas of limestone.

19.Which among the following is a long mesa that at one end slopes upward to higher terrain?

[A] Erg
[B] Inselberg
[C] Cuesta
[D] Potrero

Hide Answer

Correct Answer: D [Potrero]

Notes:
Potrero in spanish is a common land in poor condition. Potreros are long mesas that slopes upward
at one end to a higher terrain.

20.What is a closed karst depression larger than a sinkhole called?

[A] Doline
[B] Uvala
[C] Mogote
[D] Tor

Hide Answer

Correct Answer: B [Uvala]

Notes:
Karst landforms are topographic features associated with the dissolution of soluble rocks like
limestone, dolomite or gypsum. Uvalas are closed karst depressions usually larger than the sinkhole.

21.Which among the following statements is correct about Cinder Cone?

[A] It is an isolated, flat-topped underwater volcano mountain


[B] It is a cauldron-like volcanic feature formed by the emptying of a magma chamber
[C] It is a steep conical hill of loose pyroclastic fragments around a volcanic vent.
[D] It is an area of land surrounded by one or more younger lava flows

Hide Answer

Correct Answer: C [It is a steep conical hill of loose pyroclastic fragments around a volcanic vent.]
Notes:
Cinder cone is a steep conical hill of loose pyroclastic fragments around a volcanic vent. Most cinder
cones have a bowl shaped crater at the summit.

22.What is the low profile volcano usually formed almost entirely of fluid lava flows called?

[A] Caldera
[B] Cinder cone
[C] Caldera
[D] Shield volcano

Hide Answer

Correct Answer: D [Shield volcano]

Notes:
Low profile volcano usually formed almost entirely of fluid lava flows is called as Shield volcano. They
have a gentle slope. The largest and the most prominent shield volcano chain in the world is the
Hawaiin Islands.
23.The Danube delta is a part of which among the following countries?

[A] Romania
[B] Germany
[C] Sweden
[D] Russia

Hide Answer

Correct Answer: A [Romania]

Notes:
The Danube delta is the second largest river delta in Europe. The greater part of the Danube delta
lies in Romania and a small part in Ukraine.

24.Which is the longest Freshwater lake in the world?

[A] Lake Michigan


[B] Lake Erie
[C] Lake Tanganyika
[D] Lake Victoria

Hide Answer

Correct Answer: C [Lake Tanganyika]


Notes:
Lake Tanganyika is the longest freshwater lake in the world. It is second largest by volume. It is
bordered by the following countries:
Burundi
Tanzania
Zambia
Democratic Republic of the Congo

25.The Halong Bay is located in which among the following countries?

[A] Indonesia
[B] Vietnam
[C] Malaysia
[D] Myanmar

Hide Answer

Correct Answer: B [Vietnam]

Notes:
The Halong Bay is located in Vietnam. It is a UNESCO World Heritage site. It is a home to the Dau Go
Cave. The Halong Bay is famous for the limestone features.

26.Which is the world’s largest War Memorial?

[A] Great Ocean Road


[B] Big Sur
[C] Lofoten Islands
[D] Rio de Janeiro
Hide Answer

Correct Answer: A [Great Ocean Road ]

Notes:
The Great Ocean Road is a 243 kilometres coastal route in Australia. It is dedicated to the fallen
soldiers of World War I. It is the world’s largest war memorial.

27.Which among the following mountains is known as the Montage of Happy Colors?

[A] Huayna Picchu


[B] Kirkjufell
[C] Mt. Kilimanjaro
[D] Vinicunca

Hide Answer

Correct Answer: D [Vinicunca]

Notes:
Vinicunca, the most colorful and the most beautiful mountains of the world, are also known as the
Rainbow Mountain. It has hue lines of different colors due to different mineral deposits. It is located
on Peru.

28.Which is an intermediate layer beyond the ozone layer?


[A] Troposphere
[B] Thermosphere
[C] Exosphere
[D] Mesosphere

Hide Answer

Correct Answer: D [Mesosphere]

Notes:
The Mesosphere is the layer of the earth’s atmosphere which is an intermediate layer beyond the
ozone layer. It lies upto an altitude of 80 km from the earth’s surface. The temperature gradually
falls to -100°C at 80 km altitude in this layer. The meteorites burn up in this layer on entering from
the space.

29.The Great Basin Desert is located in which among the following countries?

[A] Australia
[B] Saudi Arabia
[C] Russia
[D] United States

Hide Answer

Correct Answer: D [United States]

Notes:
The Great Basin Desert is part of the Great Basin between the Sierra Nevada and the Wasatch
Range. It is located in the states of Nevada, Utah, California, Idaho and Oregon of USA. It is a
temperate desert with hot, dry summers and snowy winters.

30.Which among the following deserts is/are correctly matched?

1. Namib desert – Angola, Namibia, and South Africa

2. Lut Desert – Iraq

3. Atacama desert – United States

Select the correct code from the options given below:

[A] Only 1
[B] Only 1 & 3
[C] Only 3
[D] None of the above

Hide Answer

Correct Answer: A [Only 1 ]

Notes:
The Lut Desert also known as the Dasht-e Lut Desert is a large desert located in Iran. It is the world’s
25th-largest desert and is included on UNESCO’s World Heritage List. It covers an area of 51,800
square kilometres.

The Atacama desert in Chile, South America is the driest desert in the world. It owes its extreme
aridity to a constant temperature inversion due to the cool north-flowing Humboldt ocean current
and to the presence of the strong Pacific anticyclone.

31.Which among the following statements is correct about the landform Tafoni?

1. It is a depositional landform.

2. It is a plain where the bedrock has been subject to considerable subsurface deposition.

Select the correct code from the options given below:

[A] Only 1
[B] Only 2
[C] Both 1 & 2
[D] Neither 1 & 2

Hide Answer

Correct Answer: D [Neither 1 & 2]

Notes:
Small holes or caves on the surface of a rock formed by weathering is called as Tafoni. They are
considered as polygenetic in origin due to its formation by an interaction of complex physical and
chemical weathering.

32.The Caspian Sea is not surrounded by which among the following countries?
[A] Kazakhstan
[B] Russia
[C] Iraq
[D] Iran

Hide Answer

Correct Answer: C [Iraq]

Notes:
The Caspian sea despite its name, is one of the largest lakes in the world. It is surrounded by the
following countries:
-Kazakhstan
-Russia
-Turkmenistan
-Azerbaijan
-Iran

33.Which among the following pairs of mountain ranges and their peaks is not correctly matched?
[A] Drakensberg Mountains – Mt.Lesotho
[B] Andes Mountains – Mt. Aconcagua
[C] Atlas Mountains – Mt. Toubkal
[D] Altai Mountains – Mt. Whitney

Hide Answer

Correct Answer: D [Altai Mountains – Mt. Whitney ]

Notes:
The Altai mountain range in Central Asia, is a young folded mountain range which extends from
Kazakhstan to northern China. The Belukha Mountain is the highest peak in Altai. The Sierra Nevada
mountain ranges is in California, US. Mt.Whitney is the highest peak in Sierra Nevada. It is an
important habitat for many Red Indians.

34.Which among the following mountain ranges is located in Russia?

[A] Andes
[B] Ural
[C] Arakan Yoma
[D] Vosges

Hide Answer

Correct Answer: B [Ural ]

Notes:
The Ural Mountains in Russia, act as a boundary between Europe and Asia. Mt. Narodnaya is the
highest peak in the Urals.

35.Which cloud is also known as an Anvil cloud?

[A] Stratus
[B] Cirrus
[C] Cumulonimbus
[D] Nimbostratus

Hide Answer

Correct Answer: C [Cumulonimbus]

Notes:
Cumulonimbus clouds are dense towering vertical clouds. It’s top acquires an ‘Anvil Shape’. Hence
known as an Anvil cloud. They are associated with thunderstorms and atmospheric instability.

36.Which among the following statements is/are correct about Atlantic Ocean?

1. The Atlantic Ocean widens to the south of equator

2. It narrows towards the equator

3. It widens in the extreme north and contacts the Arctic Ocean.

Select the correct code from the options given below:

[A] Only 1
[B] Only 2
[C] Only 1 & 3
[D] Only 1 & 2

Hide Answer

Correct Answer: D [Only 1 & 2]

Notes:
The Atlantic Ocean is the second largest ocean in the world. It covers approximately 20 percent of
Earth’s surface and about 29 percent of its water surface area.
The Atlantic Ocean widens to the south of equator. It narrows down towards the equator. It narrows
down in the extreme north and contacts the Arctic Ocean.

37.The Hadalpelagic zone is associated with which among the following ocean relief features?

[A] Continental Slope


[B] Abyssal Plain
[C] Continental Rise
[D] Oceanic Trenches

Hide Answer

Correct Answer: D [Oceanic Trenches]

Notes:
Oceanic trenches, some of the deepest spots on earth, are long, narrow, relatively steep-sided
depressions in the deep sea. They occur at depths of approximately 7,300 to more than 11,000
meters. They occur in the deepest layers of the ocean, popularly known as the Hadalpelagic zone.

38.What is a detached elevation with shallow depths?

[A] Atoll
[B] Shoal
[C] Reef
[D] Trench

Hide Answer

Correct Answer: B [Shoal]

Notes:
Shoal is a detached elevation with shallow depths. It projects out of water with moderate heights.
Therefore they are dangerous for navigation and other activities.

39.Which among the following statements is/are correct about Submarine Canyons?

1. These are the flattest and smoothest regions of the world

2. The largest canyons in the world occur in the Bering Sea off Alaska.

Select the correct code from the options given below:

[A] Only 1
[B] Only 2
[C] Both 1 & 2
[D] Neither 1 & 2

Hide Answer

Correct Answer: B [Only 2]

Notes:
Submarine Canyons are deep valleys that are narrow and steep-sided. They are found cutting into
the continental slopes and shelves of the oceans. The largest canyons in the world occur in the
Bering Sea off Alaska.

40.Which among the following is/are controlling factors in the magnitude of tides?

1. Changes in position of the sun and moon in relation to the earth.

2. Irregularities in the configuration of the oceans.

3. The shape of bays and estuaries along a coastline

Select the correct code from the options given below:


[A] Only 1 & 2
[B] Only 1
[C] Only 2 & 3
[D] 1, 2 & 3

Hide Answer

Correct Answer: D [1, 2 & 3]

Notes:
Factors Controlling the Nature and Magnitude of Tides:
The movement of the moon in relation to the earth.
Changes in position of the sun and moon in relation to the earth
Uneven distribution of water over the globe.
Irregularities in the configuration of the oceans.
The shape of bays and estuaries along a coastline, etc.

41.Who among the following has given the classification of Ocean Deposits?

[A] F. Hoyle
[B] Alexander Humboldt
[C] Edwin Hubble
[D] J.T. Jenkins

Hide Answer

Correct Answer: D [J.T. Jenkins]

Notes:
Jenkins has divided marine deposits into three groups viz.:
Deep sea deposits,
Shallow water deposits, and
Littoral deposits.
He has given a detailed classification of ocean deposits.

42.Which among the following is not a cold current in the Atlantic Ocean?
[A] Falkland current
[B] Labrador current
[C] Brazilian current
[D] Benguela current

Hide Answer

Correct Answer: C [Brazilian current]

Notes:
Cold currents in the Atlantic Ocean:
Canaries Current
Labrador Current
Falkland Current
South Atlantic Drift
Benguela Current.

43.Which among the following statements is/are correct about La Nina?

1. The literal meaning of the term La Nina is a Christ child.

2. During the occurrence of this condition sea surface temperature in the central and western
Pacific rises above the normal.

Select the correct code from the options given below:

[A] Only 1
[B] Only 2
[C] Both 1 & 2
[D] Neither 1 & 2

Hide Answer
Correct Answer: D [Neither 1 & 2]

Notes:
The La Nina presents a situation that can be described as roughly opposite of the El Nino. The literal
meaning of the term La Nina is a girl child. During the occurrence of this condition sea surface
temperature in the central and western Pacific falls below the normal and this happens due to the
South Pacific Sub-tropical High becoming exceptionally strong during the summer season.

44.Which among the following statements is/are correct about the porosity of the soil?

1. A soil with high organic content tends to have low porosity.

2. Porosity is expressed as a ratio of volume of voids (pores) to the total volume of the
material.

Select the correct code from the options given below:

[A] Only 1
[B] Only 2
[C] Both 1 & 2
[D] Neither 1 & 2

Hide Answer

Correct Answer: B [Only 2]


Notes:
Porosity – The volume of water which can be held within a soil is called its porosity. It is expressed as
a ratio of volume of voids (pores) to the total volume of the material. A soil with high organic
content also tends to have high porosity.

45.Which among the following layers of soil is dominated by organic matter?

[A] O Horizon
[B] E Horizon
[C] D Horizon
[D] A Horizon

Hide Answer

Correct Answer: A [O Horizon]

Notes:
The O Horizon is the layer dominated by organic matter. Some O layers consist of undecomposed or
partially decomposed litter (such as leaves, needles, twigs, moss, and lichens). They may be on top
of either mineral or organic soils.

46.Which Blood Group is most common among the Caucasoid race?

[A] Group A
[B] Group B
[C] Group O
[D] Group AB

Hide Answer
Correct Answer: A [Group A]

Notes:
Blood type or blood groups are among the most widely studied physiological traits of human beings.
Although all blood groups are found in all racial groups, there are differences in the dominant blood
groups found among various racial
groups. Group A is most common among the Caucasoid while group B is most dominant among the
Mongoloids. Both group A and B are common among the Negro people.

47.Which among the following factors is/are correct about population growth?

1. Areas with very heavy rainfall or extreme and harsh climates have low population

2. Areas which have fertile loamy soils have dense population.

Select the correct code from the options given below:

[A] Only 1
[B] Only 2
[C] Both 1 & 2
[D] Neither 1 & 2

Hide Answer

Correct Answer: C [Both 1 & 2 ]


Notes:
Soils- Fertile soils are important for agricultural and allied activities. Therefore, areas which have
fertile loamy soils have dense population. Eg. Northern plains of India
Climate- Areas with very heavy rainfall or extreme and harsh climates have low population, for
example Mediterranean regions.Areas with a comfortable climate, where there is not much seasonal
variation attract more people.

48.Which among the following statements is/are correct about Urban Life?

1. Dependence upon agriculture for livelihood has steadily decreased.

2. Population of towns and cities has increased over the years.

Select the correct code from the options given below:

[A] Only 1
[B] Only 2
[C] Both 1 & 2
[D] Neither 1 & 2

Hide Answer

Correct Answer: C [Both 1 & 2]

Notes:
Urbanization is a universal process implying economic development and social change. Urbanization
also means, “a breakdown of traditional social institutions and values”.
Two trends are clear regarding urban life:-
– Dependence upon agriculture for livelihood has steadily decreased; and
– Population of towns and cities has increased over the years.

49.Which area was the last to witness the development of Plantation Farming?

[A] Asia
[B] South America
[C] Australi
[D] Africa

Hide Answer

Correct Answer: D [Africa]

Notes:
Africa was the last to witness development 0f plantation farming. An end to the practice of slavery
and industrial revolution in Europe led European entrepreneurs to search for additional products to
satisfy a growing demand.

50.Which among the following countries alone buys nearly one-fourth of the total manganese in the
world?

[A] Italy
[B] France
[C] Russia
[D] Spain

Hide Answer

Correct Answer: B [France]

Notes:
Manganese due to its importance in iron and steel industry and also the fact that it generally does
not occur in iron ore producing areas has considerable importance in the international trade. France
alone buys nearly one-fourth of the total manganese entering the international market

1.Which among the following states produces 60% Salt of India?

[A] Rajasthan
[B] Odisha
[C] Gujarat
[D] Maharashtra

Hide Answer

Correct Answer: C [Gujarat]

Notes:
With around 160 Lakh tons of production, India is the third largest salt producing country in the
world. Gujarat, Rajasthan, Andhra Pradesh and Tamilnadu are 4 largest salt producing states in India.

2.Pick the wrong one out:


[A] Sutlej originates from Rakas Lake (connected to Mansarovar Lake)
[B] Ravi Originates from Rohtang pass in Himalayas
[C] Jhelum originates from south eastern part of Kashmir
[D] Chenab originates from confluence of two rivers, The Chandra and Beas

Hide Answer

Correct Answer: D [Chenab originates from confluence of two rivers, The Chandra and Beas]

Notes:
The Chenab originates from the confluence of two rivers, the Chandra and the Bhaga, which
themselves originate from either side of the Bara Lacha Pass in Lahul. It is also known as the
Chandrabhaga in Himachal Pradesh. It runs parallel to the Pir Panjal Range in the north-westerly
direction, and cuts through the range near Kishtwar. It enters the plains of Punjab near Akhnur and
is later joined by the Jhelum. It is further joined by the Ravi and the Sutlej in Pakistan

3.Which among the following is the chief crop of Deccan Plateau?

[A] Wheat
[B] ground nut
[C] Pulses
[D] Cotton

Hide Answer

Correct Answer: D [Cotton]

Notes:
The regur soil of Deccan plateau is suitable for cotton cultivation and so cotton is the chief crop of
Deccan plateau. The black color of regur soil is due to its iron content which has derived from
plutonic lava materials.

4.Sundarbans in India is a fine example of which among the following kind of forests?
[A] Dry Forests
[B] Deciduous Forests
[C] Tidal Forests
[D] Wet Deciduous Forests

Hide Answer

Correct Answer: C [Tidal Forests ]

Notes:
Tidal forests grows mainly in the deltaic regions of the Ganga, Mahanadi, Godavari and Krishna
which are flooded by tides and high sea waves. Mangrove is the representative of this type of
vegetation. Sundari is the typical tree of tidal forests. It is found in abundance in the lower Ganga
delta of West Bengal. This is the reason why it is popularly known as Sunderban. It is known for its
hard and durable timber.

5.Which is the only state in India producing muga silk ?

[A] Assam
[B] Bihar
[C] Orissa
[D] West Bengal

Hide Answer

Correct Answer: A [Assam]

6.Which among the following type of soil has the largest area covered in India?

[A] Alluvial
[B] Black
[C] Red
[D] Laterite

Hide Answer

Correct Answer: A [Alluvial]

Notes:
Alluvial soils, the depositional soils transported by rivers, are the predominant type of soil in the
northern plains of the country, widespread in the Ganga plains and the river valleys. These soils
cover about 40 per cent of the total area of India and have largest area covered in India.

7.Which among the following Gulfs is noted for the extreme rise and fall of its tides, making it ideal
location for Tidal energy ?
[A] Gulf of Mannar
[B] Gulf of Khambat
[C] Gulf of Kutch
[D] Both 2 and 3

Hide Answer

Correct Answer: D [Both 2 and 3]

8.Which among the following was steel plant of India is sometimes called India’s First Swadeshi Steel
Plant?

[A] Bengal Iron Works Company


[B] TISCO
[C] IISCO
[D] Bokaro Steel Plant

Hide Answer

Correct Answer: D [Bokaro Steel Plant]

9.Arrange the following states of India in descending order of their geographical area:

1. Andhra Pradesh

2. Madhya Pradesh

3. Maharashtra

4. Uttar Pradesh
Choose the correct option from the codes given below:

[A] 1, 3, 2, 4
[B] 2, 1, 4, 3
[C] 2, 3, 4, 1
[D] 1, 4, 3, 2

Hide Answer

Correct Answer: C [2, 3, 4, 1]

Notes:
Correct descending order- Madhya Pradesh, Maharashtra, Uttar Pradesh, Andhra Pradesh.

10.How many metropolitan cities are there in India?

[A] 8
[B] 9
[C] 13
[D] 23

Hide Answer

Correct Answer: D [23]

Notes:
The 74th Amendment to the Indian Constitution defines a metropolitan area as an area having a
population of 10 Lakh or 1 Million or more, comprised in one or more districts and consisting of two
or more Municipalities or Panchayats or other contiguous areas, specified by the Governor by public
notification. At present there are around 23 metropolitan areas viz. Ahmedabad, Bangalore,
Chennai, Coimbatore, Gorakhpur, Hyderabad, Jaipur, Jodhpur, Kanpur, Kochi, Kolkata, Kozhikode,
Madurai, Mumbai, Nagpur, National Capital Region, Patna, Pune, Raipur, Salem, Surat,
Thiruvananthapuram and Visakhapatnam.
1.Which among the following is/ are correct about the Great plains of India?
1. It is also known as the Granary of India
2. The rivers of this plain in perennial in nature
3. The groundwater level is very high in this region
4. This area supports around 40% of the total population of India
Choose the correct option from the codes given below :

[A] 1 & 2
[B] 1, 3 & 4
[C] 2, 3 & 4
[D] 1, 2, 3 & 4

Hide Answer

Correct Answer: D [1, 2, 3 & 4]

Notes:
The Great plains of India (Northern) is also known as the Granary of India. The rivers of this plain are
perennial in nature. The groundwater level is very high in this region. This area supports around 40%
of the total population of India.
12.The vertical rock face known as Hillary step is located in which of the following mountain peak?

[A] Mt. Everest


[B] Kanchenjunga
[C] Nanda Devi
[D] Trishul

Hide Answer

Correct Answer: A [Mt. Everest]

Notes:
Hillary step is located in the Mt. Everest. It is named after Sir Edmund Hillary (The first man to climb
the mountain).

13.Which one among the following passes connects Lhasa with Ladakh?

[A] Khyber
[B] Burzil
[C] Babusar
[D] Lanak La

Hide Answer

Correct Answer: D [Lanak La]


Notes:
Lanak La pass connects Ladakh with Lhasa. It is located at an elevation of 5466 meters in Aksai-China
(Ladakh).

14.Which among the following passes are located in the state of Uttarakhand?
1. Lipu Lekh
2. Thang La
3. Niti Pass
4. Muling La
Choose the correct option from the codes given below :

[A] 1 & 2
[B] 1, 3 & 4
[C] 2, 3 & 4
[D] 1, 2, 3 & 4

Hide Answer

Correct Answer: D [1, 2, 3 & 4]

Notes:
The following pass are located in Uttarakhand- Lipu Lekh, Thang La, Niti Pass, Muling La, Mana Pass,
Mangsha Dhura Pass, Traill’s Pass.

15.Rohtang Pass connects which of the following places?

[A] Kullu valley with Lahaul and Spiti


[B] Pindari valley with Milam valley
[C] Uttarakhand with Tibet
[D] Sikkim with Lhasa

Hide Answer

Correct Answer: A [Kullu valley with Lahaul and Spiti]

Notes:
Rohtang Pass is located in the Pir Panjal range at an elevation of 3,980 meters. It connects the Kullu
valley with Lahaul and Spiti.

16.Arrange the following Hill ranges as per their geological periods:


1. Deccan plateau
2. Himalayas
3. Eastern Ghat
4. Aravalli
Choose the correct option from the codes given below:

[A] 1, 2, 3, 4
[B] 4, 3, 1, 2
[C] 4, 1, 2, 3
[D] 4, 2, 3, 2

Hide Answer

Correct Answer: B [4, 3, 1, 2]

Notes:
The correct order of the Hill ranges as per their geological periods Aravalli, Eastern Ghats, Deccan
Plateau, Himalayas.

17.That desert is located in which side of Aravalli range?

[A] Eastern
[B] Nothern
[C] Southern
[D] Western

Hide Answer

Correct Answer: A [Eastern]

Notes:
That desert is located in the eastern and the Noth eastern side of Aravalli range of the state of
Rajasthan.

18.Which of the following is the new name of the Ross island?


[A] Mahatma Gandhi Island
[B] Abdul Kalam Island
[C] Jawaharlal Nehru Island
[D] Netaji Subhash Chandra Bose Island

Hide Answer
Correct Answer: D [Netaji Subhash Chandra Bose Island]

Notes:
The new name of the Ross island is Netaji Subhash Chandra Bose Island. The new name was given in
the year 2018. This island is a part of the South Andaman administrative area.

19.Which of the following subgroup of Lakshadweep island was previously known as Cannanore
Islands?

[A] Aminidivi Islands


[B] Laccadive Islands
[C] Minicoy Island
[D] None of the above

Hide Answer

Correct Answer: B [Laccadive Islands]

Notes:
The Laccadive subgroup was previously known as Cannanore Islands. The only airport in
Lakshadweep, Agatti Aerodrome is situated in one of the subgroups of Laccadive Agatti island.

20.Which one among the following is the coral group of islands situated in India?

[A] Nicobar
[B] Lakshadweep
[C] Minicoy
[D] Andaman

Hide Answer

Correct Answer: B [Lakshadweep]

Notes:
Lakshadweep is about 220 to 440 Kms away from the Kerala coast. It is a group of 36 coral islands.
All these islands have coral reefs

21.Which of the following is the first beach of India to get Blue Flag certification?

[A] Konark Beach


[B] Puri Beach
[C] Chandrabhaga Beach
[D] Satpada Beach

Hide Answer

Correct Answer: C [Chandrabhaga Beach]

Notes:
The tag of Blue Flag or Blue Flag certification is given on the basis of the environment-friendly clean
conditions of beaches, and facilities of international standards for tourists. Chandrabhaga Beach is
the first beach in India to get Blue Flag certification. It is located in the state of Odisha.

22.Campbell Bay is located in which of the following places of India?


[A] Little Andaman
[B] Little Nicobar
[C] South Andaman
[D] Great Nicobar

Hide Answer

Correct Answer: D [Great Nicobar]

Notes:
Campbell Bay is located in Great Nicobar island. The southernmost point of India, Indira Point is
located here.

23.Which among the following about the eastern coast of India are correct?
1. It is formed by emergence
2. Eastern coast face most of the cyclonic storms
3. It is located on the Bay of Bengal
4. Rainfall is comparatively low
Choose the correct option from the codes given below :
[A] 1 & 3
[B] 2 & 4
[C] 1, 2 & 4
[D] 1, 2, 3 & 4

Hide Answer

Correct Answer: D [1, 2, 3 & 4]

Notes:
The Eastern coast of India is located on the Bay of Bengal. It is formed by emergence. Eastern coast
faces most of the cyclonic storms. Rainfall is comparatively low.

24.Which among the following plateau have a famous gold field?

[A] Karbi Anglong Plateau


[B] Karbi Meghalaya Plateau
[C] Kaas Plateau
[D] Kolar Plateau

Hide Answer

Correct Answer: D [Kolar Plateau]

Notes:
The famous Kolar Gold Field is located at Kolar Plateau. It is situated in the state of Karnataka. The
region is also famous for woolen blankets, leather goods, hand-loomed silk, etc.

25.The Deranged pattern of drainage is found in which of the following region of India?
[A] Chotanagpur Plateau
[B] Southern Peninsular Region
[C] Desert region
[D] Valleys of Karakoram
Hide Answer

Correct Answer: D [Valleys of Karakoram]

Notes:
The Deranged pattern of drainage is found in the Valleys of Karakoram. It is the region that is
evacuated by the ice sheet.

26.Yamuna river was the tributary of which of the following rivers before it became a tributary of the
Ganga river?

[A] Mahanadi
[B] Godavari
[C] Indus
[D] Tapi

Hide Answer

Correct Answer: C [Indus]

Notes:
Previously Yamuna river had a course towards the southwest direction. It was a tributary of the
Indus river.

27.The river Alakananda is originated from which of the following glaciers?


[A] Satopanth Glacier
[B] Gangotri Glacier
[C] Siachen Glacier
[D] Pindari Glacier

Hide Answer

Correct Answer: A [Satopanth Glacier]

Notes:
The river Alakananda is originated from Satopanth Glacier. It is located in the state of Uttarakhand. It
joins the Bhagirathi river at Devprayag.

28.Tiau river is located on the boundary of which of the following pairs of country?

[A] Indian & Nepal


[B] India & Bhutan
[C] India & Myanmar
[D] India and Pakistan

Hide Answer

Correct Answer: C [India & Myanmar]

Notes:
Tiau river (159 km) is located on the boundary of India & Myanmar. It is located in the Indian state of
Mizoram.

29.The snow-covered Ghepan Lake is located in which of the following places?


[A] Sikkim
[B] Uttarakhand
[C] Himachal Pradesh
[D] Jammu & Kashmir

Hide Answer

Correct Answer: C [Himachal Pradesh]

Notes:
The Ghepan Lake is located in Himachal Pradesh.

30.‘Govind Ballabh Sagar’ is located in which of the following states?

[A] Uttar Pradesh


[B] Orissa
[C] West Bengal
[D] Uttarakhand

Hide Answer

Correct Answer: A [Uttar Pradesh]

Notes:
Govind Ballabh Sagar is situated in Uttar Pradesh

31.Kaziranga national park is famous for which of the following species?

[A] Lion
[B] Crocodile
[C] Rhinoceros
[D] Tiger

Hide Answer

Correct Answer: C [Rhinoceros]

Notes:
Kaziranga national park is famous for its horn Rhinoceros.

32.Which of the following plant is known for it biodiesel property?

[A] Sunflower
[B] Jatropha
[C] Maize
[D] Pongamia

Hide Answer

Correct Answer: B [Jatropha]

Notes:
Jatropha is a genus of flowering plants belongs to the family, Euphorbiaceae.
33.The diverse flora and fauna of the planet are under great threat mainly due to which of the
following reason?

[A] Jhum cultivation


[B] Modern method of agriculture
[C] Insensitivity to our environment
[D] Commercial agriculture

Hide Answer

Correct Answer: C [Insensitivity to our environment]

Notes:
Insensitivity to our environment is the main reason for the diverse flora and fauna of the planet are
under great threat.

34.Malaj Khand of Madhya Pradesh is associated with which of the following mineral production?

[A] Copper
[B] Lignite
[C] Bauxite
[D] Iron ore

Hide Answer

Correct Answer: A [Copper]

Notes:
Malajkhand is famous for its copper mine.

35.Which of the following state is the largest Mica producing state of India?

[A] Odisha
[B] Andhra Pradesh
[C] Karnataka
[D] Rajasthan

Hide Answer

Correct Answer: B [Andhra Pradesh]

Notes:
Andhra Pradesh is the largest Mica producing state of India. It accounts for about 71% of total Mica
production in India. Some of the mining centers are Nellore, Khamma, Gudur mines.

36.Which of the following groups of states accounts for about 90% of the annual coal production in
India?
[A] Orissa, Madhya Pradesh and Rajasthan
[B] Jharkhand, Orissa and West Bengal
[C] Madhya Pradesh, Tamil Nadu and West Bengal
[D] Jharkhand, Orissa and Madhya Pradesh

Hide Answer

Correct Answer: B [Jharkhand, Orissa and West Bengal]


Notes:
West Bengal, Jharkhand and Orissa accounts for about 90% of the annual coal production in India.

37.Kalakot tertiary coalfield is located in which of the following places of India?

[A] Himalayan mountain region


[B] Cardamom hills of Kerala
[C] Damodar river basin of Jharkhand and West Bengal
[D] Brahmaputra river basin Assam

Hide Answer

Correct Answer: A [Himalayan mountain region]

Notes:
Kalakot tertiary coalfield is located in the Himalayan mountain region.

38.Ukai dam is situated across which of the following river?

[A] Narmada
[B] Tapi
[C] Mahanadi
[D] Krishna

Hide Answer

Correct Answer: B [Tapi]

Notes:
Ukai dam is situated across the Tapi river. The main purpose of constructing this dam was to harness
the Tapi river. It is located in the state of Gujarat.

39.Which among the following is the oldest nuclear power station in India?
[A] Kaiga Nuclear Power Plant
[B] Narora Atomic Power Station
[C] Tarapur Atomic Power Station
[D] Kakrapar Atomic Power Station

Hide Answer

Correct Answer: C [Tarapur Atomic Power Station]

Notes:
Tarapur Atomic Power Station is the oldest nuclear power station in India. It is located in the state of
Maharashtra. It was established in the year 1969.

40.In the year 1963, two boiling water reactor (BWR) units were installed in Tarapur Atomic Power
Station by the agreement with which of the following foreign countries?

[A] USSR
[B] USA
[C] UK
[D] Canada

Hide Answer
Correct Answer: B [USA]

Notes:
In the year 1963, two boiling water reactor (BWR) units were installed in Tarapur Atomic Power
Station by the agreement of India, USA and International Atomic Energy Agency (IAEA). The
commercial operation was started in 1969.

41.Which of the following statements is/ are correct?


1. Jhum cultivation is practiced in the North-Eastern State of India
2. Jhum cultivation is a process of afforestation
Choose the correct option from the codes given below:

[A] 1 only
[B] 1 & 2
[C] 2 only
[D] None of the above

Hide Answer

Correct Answer: A [1 only]

Notes:
Jhum (Shifting) cultivation is a type of primitive practice of cultivation which is adopted in the North-
Eastern State of India.

42.Which among the following statements are correct?


1. The leading consumer of the fertilizer is Punjab
2. In the areas of dry farming, the fertilizer consumption is very low
3. The largest number of cattle are found in India
Choose the correct option from the codes given below :

[A] 1 & 2
[B] 1 & 3
[C] 2 & 3
[D] 1, 2 & 3

Hide Answer

Correct Answer: D [1, 2 & 3]

Notes:
The leading consumer of the fertilizer is Punjab (175kg/ha). In the areas of dry farming, the fertilizer
consumption is very low. Largest number of cattle are found in India.
43.Arrange the following states as per the descending order of their production of cotton textile
1. Tamil Nadu
2. Maharashtra
3. Gujarat
4. Uttar Pradesh
Choose the correct option from the codes given below :

[A] 2, 1, 3, 4
[B] 2, 3, 1, 4
[C] 3, 2, 1, 4
[D] 3, 1, 2, 4

Hide Answer

Correct Answer: B [2, 3, 1, 4]

Notes:
The correct descending order- Maharashtra, Gujarat, Tamil Nadu, Uttar Pradesh.

44.Which among the following matches of the Aluminium industry and their locations are correct?
1. HINDALCO- Renukoot
2. MALCO- Mettur
3. BALCO- Korba
4. NALCO- Koraput
Choose the correct option from the codes given below :

[A] 1 & 2
[B] 1, 2 & 3
[C] 2, 3 & 4
[D] 1, 2, 3 & 4

Hide Answer

Correct Answer: D [1, 2, 3 & 4]

Notes:
The correct matches- The Hindustan Aluminium Corporation Ltd. (HINDALCO)- Renukoot. The
Madras Aluminium Corporation Ltd. (MALCO)- Mettur. The Bharat Aluminium Company Ltd.
(BALCO)- Korba. The National Aluminium Company Ltd. (NALCO)- Koraput.

45.The maximum length of NH-44 lies in which of the following states?

[A] Rajasthan
[B] Madhya Pradesh
[C] Telangana
[D] Tamil Nadu

Hide Answer

Correct Answer: D [Tamil Nadu]

Notes:
The maximum length of NH-44 lies in the state of Tamil Nadu. The total length of NH-44 in Tamil
Nadu is 627 kilometers.

46.The headquarter of Hindustan Shipyard Limited is situated in which of the following places of
India?

[A] Mumbai
[B] Goa
[C] Visakhapatnam
[D] Kochin

Hide Answer
Correct Answer: C [Visakhapatnam]

Notes:
The headquarter of Hindustan Shipyard Limited is situated in Visakhapatnam. The former name of
the organization was Scindia Shipyard. The nuclear-powered Arihant class submarine was developed
here.

47.Arrange the following ports of Eastern coast of India from North to South
1. Haldia Port
2. Vizag Port
3. Chennai Port
4. Kolkata Port
Choose the correct option from the codes given below :

[A] 4, 3, 1, 2
[B] 2, 1, 4, 3
[C] 4, 1, 2, 3
[D] 2, 4, 1, 3

Hide Answer

Correct Answer: C [4, 1, 2, 3]

Notes:
The correct order from North to South- Kolkata Port (West Bengal), Haldia Port (West Bengal), Vizag
Port (Andhra Pradesh), Chennai Port (Tamil Nadu).

48.As per the census conducted in 2011, which of the following state has the lowest population
density?

[A] Kerala
[B] Uttar Pradesh
[C] West Bengal
[D] Bihar

Hide Answer

Correct Answer: B [Uttar Pradesh]

Notes:
Bihar (1106), West Bengal (1029), Kerala (859), Uttar Pradesh (828).

49.The Jain population in India is highest in which of the following states?

[A] Maharashtra
[B] Gujarat
[C] Rajasthan
[D] Madhya Pradesh

Hide Answer

Correct Answer: A [Maharashtra]


Notes:
The Jain community constitutes only 0.4% of the total population. In the state of Maharashtra
(1,400,349) the number of Jains are highest.

50.What is the annual number of live births per 1,000 people called?

[A] General Fertility rate


[B] Crude birth rate
[C] Total fertility rate
[D] Gross reproduction rate

Hide Answer

Correct Answer: B [Crude birth rate]

Notes:
Demography is a statistical study of human population. It studies a variety of variables related to
population like size, growth, distribution, density, composition and their spatial and temporal
variations.
Crude Birth Rate: The annual number of live births per 1,000 people

1.The Bhimbetka rock shelters are an archaeological site of the Palaeolithic period. It is located in
which state?

[A] Gujarat
[B] Madhya Pradesh
[C] Maharashtra
[D] Jharkhand

Hide Answer

Correct Answer: B [Madhya Pradesh]

Notes:
The Bhimbetka rock shelters are an archaeological site of the Palaeolithic, exhibiting the earliest
traces of human life on the Indian Subcontinent. It is located in the Raisen District in the Indian state
of Madhya Pradesh, near Abdullaganj town and inside the Ratapani Wildlife Sanctuary.

2.The first novel written by Sunil Gangopadhyay is :

[A] Athmo Prakash (Self-Revelation)


[B] Pratham Alo (First Light)
[C] Purba Pachim (East and West)
[D] Shei Shomay (Those Days)

Hide Answer

Correct Answer: A [Athmo Prakash (Self-Revelation)]

Notes:
Starting off his six-decade literary career as a bohemian poet and editor of Kritibas, a monthly poetry
magazine, Sunil Gangopadhyay wrote his first novel, Athmo Prakash (Self-Revelation), at the behest
of the editor of the hugely popular periodical Desh for its special Durga Puja edition. Two of the most
critically acclaimed films of legendary filmmaker Satyajit Ray — Pratidwandi and Aranyer Din Ratri —
were based on novels written by him.

3.According to the beliefs of the Pasupata sect, Siva is the lord of Pasu. Pasu here refers to __?

[A] Lion
[B] Bull
[C] Parvati
[D] Jiva

Hide Answer

Correct Answer: D [Jiva]

Notes:
The Pasupata was a Saivite sect associated with Siva in his aspect of the “Herdsman” (pasu, “animal”
and pati, “lord”). The devotees of Pashupati Siva are called Pashupatas.

4.Who among the following has written the famous Bangla book “Agni Vina”?

[A] Rabindra Nath Tagore


[B] Kazi Nazrul Islam
[C] Bankim Chandra Chattopadhyay
[D] Sarat Chandra Chattopadhyay

Hide Answer

Correct Answer: B [Kazi Nazrul Islam]

Notes:
Kazi Nazrul Islam was a famous Bengali poet, musician and revolutionary who pioneered poetic
works invoking powerful spiritual rebellion against fascism and oppression. Accomplishing a plethora
of highly praised works through his life, Nazrul is officially known as the ‘National poet of
Bangladesh’ and celebrated in India.

5.Bishnupur which is famous for terracotta temples is located in which state of India?
[A] Orissa
[B] West Bengal
[C] Chhattisgarh
[D] Bihar

Hide Answer

Correct Answer: B [West Bengal]

Notes:
Bishnupur is located in West Bengal. It’s known for its Hindu temples made from local terracotta,
such as the pyramid-shaped Rasmancha, established around 1600.

6.The following places represent the contemporary 4 Dhamas of Hindu Pilgrimage in India?

[A] Badrinath -Kedarnath – Amarnath – Vaishnodevi


[B] Badrinath – Kedarnatrh – Gangotri – Yamunotri
[C] Badrinath – Kedarnath – Rameshwaram – Puri
[D] Badrinath -Rameshwaram -Puri -Dwarka

Hide Answer

Correct Answer: D [Badrinath -Rameshwaram -Puri -Dwarka]

Notes:
Badrinath -Rameshwaram -Puri -Dwarka and they represent 4 corners of the country

7.In which of the following states of India , a festival called Chapchar Kut is celebrated in March ?

[A] Assam
[B] Mizoram
[C] Karnataka
[D] Sikkim

Hide Answer

Correct Answer: B [Mizoram]

Notes:
The Chapchar Kut is a festival of Mizoram, India. It is celebrated during March after completion of
their most arduous task of jhum operation i.e., jungle-clearing (clearing of the remnants of burning).
It is a spring festival celebrated with great favour and gaiety.

8.How many tirthankara are the part of Jain Dharma tradition?

[A] Twenty-Five
[B] Twenty-Four
[C] Twenty
[D] Twenty-Two

Hide Answer

Correct Answer: B [Twenty-Four]


Notes:
In total, there have been twenty four tirthankaras in Jainism. Jains trace their spiritual ideas to these
twenty-four leaders or tirthankaras, with the first being Rishabhanatha, who according to Jain
tradition lived millions of years ago, was the first tirthankar. The twenty-third tirthankara
was Parshvanatha in 900 BCE, and the twenty-fourth tirthankara the Mahavira around 500 BCE who
was the last tirthanka in Jain tradition.

9.During the ‘Sao Joao’ festivities, Goans present which of the followings to each other?

[A] Flowers
[B] Fruits
[C] Beer
[D] Fish

Hide Answer

Correct Answer: B [Fruits]


Notes:
Sao Joao is a Catholic festival celebrated in Goa in an unusual manner, with people leaping into, and
swimming in, wells, streams and ponds as a form of tribute to St. John the Baptist and then claim a
reward of the seasonal jackfruit and pineapples from friends and relatives. It occurs on June 24 every
year.

10.Rani-ki-Vav (the Queen’s Stepwell), which is in the UNESCO World Heritage List, is located in
which state?

[A] Gujarat
[B] Madhya Pradesh
[C] Rajasthan
[D] Uttar Pradesh

Hide Answer

Correct Answer: A [Gujarat]

Notes:
Rani ki Vav is a stepwell situated in Patan, Gujarat.It is located on the banks of Saraswati river. Its
construction is attributed to Udayamati, the queen of 11th-century Chalukya king Bhima I. It has
been listed as one of UNESCO’s World Heritage Sites since 2014

11.Name the National Fruit of India.

[A] Apple
[B] Grapes
[C] Mango
[D] Pineapple

Hide Answer

Correct Answer: C [Mango]

Notes:
Mango (Mangifera indica), popularly known as the ‘King of Fruits’, is the National Fruit of India. It is
one of the most widely cultivated fruits of the tropical world. There are over 100 varieties of mangos
found in India, in different sizes, shapes and colors. As the national fruit of India it represents
prosperity, abundance and richness in favor of the country’s image.

12.31st Surajkund International Crafts Mela has begun at which of the following city?

[A] Faridabad
[B] Jaipur
[C] New Delhi
[D] Jaisalmer

Hide Answer

Correct Answer: A [Faridabad]

Notes:
The 31st edition of Surajkund International Crafts Mela held in Faridabad, Haryana. Over 20
countries are participating in the event with Egypt as a partner country this year and Jharkhand as
the theme state. Every year, one country becomes the partner country and an Indian state is
selected to set the theme of the mela.

13.‘Soorya Festival’ is associated to which of the following cities?

[A] Thiruvananthapuram
[B] Jaipur
[C] Chennai
[D] Hyderabad

Hide Answer

Correct Answer: A [Thiruvananthapuram]

Notes:
The Soorya Festival is one of the world’s largest arts festivals, which in 2017 was located in
thiruvanantapuram and spanned 111 days and featured 2,000 artists from across India. It is the
biggest art and cultural event in Kerala.

14.‘Mysuru Peta’ is a traditional attire worn on which part of body?

[A] Shoulder
[B] Wrist
[C] Waist
[D] Head

Hide Answer

Correct Answer: D [Head]

Notes:
The people of the Mysuru district and the Kodagu district feel proud to wear turbans called Mysuru
peta. Made with colourful pieces of cotton scarfs bestrewn in silk and zari, the Mysore ‘peta’
(turban) is a classic headgear that draws its roots from the erstwhile emperors of Mysore, called
Wodeyars. In Kodagu district, it is part of the traditional dress worn on special occasions such as
weddings. Menwear a turban during religious ceremonies to portray respect and reverence towards
the Supreme Being.

15.Which of the following classical dances and their places of origin is incorrectly matched?

[A] Kuchipudi- Odisha


[B] Bharatanatym- Tamilnadu
[C] Sattriya- Assam
[D] Mohiniattam- Kerala

Hide Answer

Correct Answer: A [Kuchipudi- Odisha]


Notes:
Kuchipudi is one of the eleven major Indian classical dances. It originated in a village named
Kuchipudi in the Indian state of Andhra Pradesh. Kuchipudi is a dance-drama performance, with its
roots in the ancient Hindu Sanskrit text of Natya Shastra.
16.The ____ dance form originated as a tribal dance in the forests of Odisha in the 18th century and
attained the status of a martial art-based dance form in the 19th century:

[A] Chhobia
[B] Bhawai
[C] Chhau
[D] Kalaripayattu

Hide Answer

Correct Answer: C [Chhau]

Notes:
The Chhau dance of Odisha originated as a tribal dance in the forests of Odisha in the 18th century.
It is a semi classical Indian dance with martial, tribal and folk traditions. The dance ranges from
celebrating martial arts, acrobatics and athletics performed in festive themes of a folk dance, to a
structured dance with religious themes.

17.Which philosophy is famous by the names of “Aulukya” or “Pashupat” philosophy?


[A] Samkhya
[B] Vaisheshika
[C] Mīmāṃsā
[D] Nyaya

Hide Answer

Correct Answer: B [Vaisheshika]

Notes:
Vaisheshika philosophy is famous by the names of “Aulukya” or “Pashupat” philosophy. Its original
originator is Rishi Kanad. This philosophy is very similar to the philosophy of justice, but in fact, it is
an independent materialistic philosophy. In this philosophy, the true nature of religion has been
described. In this, the means of worldly progress and self-reliant accomplishment have been
considered as a religion.

18.Which village of Madhubani district is the main center of Madhubani painting?

[A] Banka Village


[B] Arwal Village
[C] Jitwarpur Village
[D] None of these

Hide Answer

Correct Answer: C [Jitwarpur Village]

Notes:
Jitwarpur Village of Madhubani district is the main center of Madhubani painting. Madhubani
painting is a style of painting, prevalent in the Mithila region of Bihar state. Themes revolve around
Hindu gods and mythology, with scenes from the royal court and social events such as weddings.
Normally no space is left blank; The gaps are filled with pictures of flowers, animals, birds, and even
geometric designs. In these paintings, artists use leaves, herbs, and flowers to create the color used
to draw the paintings.
19.In which sculpture style, Tribhanga posture has been used extensively?

[A] Gandhara style


[B] Mathura style
[C] Amaravati style
[D] All of the above

Hide Answer

Correct Answer: C [Amaravati style]

Notes:
Amaravati sculpture is an ancient sculpture style, dating back to the 2nd century BC in south-eastern
India. from the third century, BC flourished during the reign of the Satavahana dynasty. It is known
for its magnificent embossed frescoes, one of the best examples of narrative sculpture in the world.
This sculpture had gained a lot of fame due to its uniqueness and excellent artistic skills. Tribhanga
posture has been used extensively in the sculptures of this style.

20.Votive Stupa is related to which life of Buddha?


[A] Made to commemorate events in the life of the Buddha or his disciples
[B] to commemorate visits
[C] to symbolise aspects of Buddhist theology
[D] All of the above

Hide Answer

Correct Answer: B [to commemorate visits]

Notes:
Votive stupas, usually built to commemorate visits to the site of major stupas or to gain spiritual
benefits, are regularly visited.

21.Dandia is a form of dance associated with which of the following state?

[A] Punjab
[B] Gujarat
[C] Haryana
[D] Rajasthan

Hide Answer

Correct Answer: B [Gujarat]

Notes:
Dandiya is the traditional folk-dance of Gujarat and Rajasthan. Along with Garba, it is the featured
dance of Navratri evenings. Originating as devotional Garba dances, which were always performed in
Durga’s honour, this dance form is actually the staging of a mockfight between the Goddess and
Mahishasura. The sticks of the dance represent the swords of Durga.

22.Sikkil Mala Chandrasekhar is associated with which instruments?

[A] Sitar
[B] Flute
[C] Veena
[D] Tabla

Hide Answer

Correct Answer: B [Flute]

Notes:
Sikkil Mala Chandrasekhar is a noted South Indian carnatic flautist. In 2006 she received “Shanmukha
Sangeetha Shiromani” award. In 2003 she received the title of “Venugaana Sironmani”. In 2002 she
received Nadakkanal award. In 2001 she received the Music Academy’s Award.

23.Abul Hasan was the court painter of which ruler?

[A] Jahangir
[B] Aurangzeb
[C] Shah Jahan
[D] Akbar

Hide Answer

Correct Answer: A [Jahangir]

Notes:
The prominent painters of Jahangir’s time were ‘Farukh Beg’, ‘Daulat’, ‘Manohar’, ‘Bisandas’,
‘Mansoor’ and Abul Hasan. ‘Farukh Beg’ painted the portrait of Sultan ‘Adil Shah’, the ruler of
Bijapur. The form of Mughal painting is a mixture of Persian and Indian styles as well as a
combination of various cultural aspects.

24.Mahasundari Devi is a famous artist of which folk painting?

[A] Pithora
[B] Madhubani
[C] Pattchitra
[D] Kalaam

Hide Answer

Correct Answer: B [Madhubani]

Notes:
Mahasundari Devi is a famous artist of Madhubani folk painting. Madhubani painting is a style of
painting, prevalent in the Mithila region of Bihar state. Themes revolve around Hindu gods and
mythology, with scenes from the royal court and social events such as weddings. Normally no space
is left blank; The gaps are filled with pictures of flowers, animals, birds, and even geometric designs.
In these paintings, artists use leaves, herbs, and flowers to create the color used to draw the
paintings.

25.Through which sculpture, for the first time, the physical and emotional expressions came closer?
[A] Gandhara style
[B] Mathura style
[C] Amaravati style
[D] All of the above
Hide Answer

Correct Answer: C [Amaravati style]

Notes:
For the first time, physical and emotional expressions came closer through Amaravati sculpture. The
minimal number of ornaments in Amaravati sculpture indicates a low attraction for ornaments
among women.

26.Votive Stupa is related to which life of Buddha?

[A] Made to commemorate events in the life of the Buddha or his disciples
[B] to commemorate visits
[C] to symbolise aspects of Buddhist theology
[D] All of the above

Hide Answer

Correct Answer: B [to commemorate visits]

Notes:
Votive stupas, usually built to commemorate visits to the site of major stupas or to gain spiritual
benefits, are regularly visited.

27.The Panmalai temple of Arcot is related to which style of Pallava art?


[A] Mahendra style
[B] Maamall style
[C] Raj Singh style
[D] None of these

Hide Answer

Correct Answer: C [Raj Singh style]

Notes:
Examples of Rajasimha style are the coastal temple at Mahabalipuram, the Panmalai temple at
Arcot, the Kailasanatha at Kanchi, and the temple at Baikuntha Perumal, etc. In the last and
important ‘Raj Singh style’ of the Pallava period, instead of rock-cut architecture, the construction of
temples started with stone, brick, etc.

28.When was the foundation stone of the Victoria Memorial laid?

[A] 1900
[B] 1903
[C] 1906
[D] 1919

Hide Answer

Correct Answer: C [1906]

Notes:
The Victoria Memorial is a large marble building in Kolkata, West Bengal, India, built between 1906
and 1921. It is dedicated to the memory of Queen Victoria (1819–1901). It is now a museum and
tourist destination under the aegis. Ministry of Culture. The memorial is situated on the Maidan
(Maidan) on the banks of the Hooghly River.

29.Which is the classical dance of Kerala?

[A] Kolattam
[B] Mahasu
[C] Mohiniyatam
[D] Kuchipudi

Hide Answer

Correct Answer: C [Mohiniyatam]

Notes:
Mohiniyattam is a semi-classical dance performed by the women of Kerala who are believed to be
older than Kathakali. Literally considered to be the center of dance, the magical Mohiniattam was
mainly performed in the temples of Kerala. It is also believed to be the successor of Devadasi dance
heritage such as Bharata Natyam, Kuchipudi, and Odissi.

30.Vilasini Natyam is a folk dance of which state?

[A] Tamil Nadu


[B] Kerala
[C] Andhra Pradesh
[D] Karnataka

Hide Answer

Correct Answer: C [Andhra Pradesh]

Notes:
This is a dance tradition of Devadasis – in Andhra Pradesh. It was revived by the anti-dev dasi act and
fortunately a few remaining dancers, it was on the verge of extinction, it is yet to receive the status
of Indian classical dance.

31.Sangeet Natak Akademi was established in?

[A] 1944
[B] 1953
[C] 1938
[D] 1987

Hide Answer

Correct Answer: B [1953]

Notes:
The Sangeet Natak Akademi, the highest institution in the field of performing arts in the country,
was established in 1953 to preserve and promote the vast imperfect heritage of India’s diverse
culture expressed in the forms of music, dance and drama. The management of the academy is
entrusted to its General Counsel. The President of the Academy is appointed by the President of
India for a term of five years. The functions of the Academy are set out in the Memorandum of
Association of the Academy, approved at the time of its registration as a Society on 11 September
1961. The registered office of the Academy is at Ravindra Bhavan, 35 Feroze Shah Road, New Delhi.
The Sangeet Natak Akademi is an autonomous body of the Ministry of Culture, Government of India.

32.In which state of India Bommalattam puppet show is famous?

[A] Assam
[B] Tamil Nadu
[C] Odisha
[D] Maharastra

Hide Answer

Correct Answer: B [Tamil Nadu]

Notes:
Tamil Nadu Puppetry, also known as puppetry, combines the techniques of both rod and string
puppetry. They were made of wood and the strings were tied to an iron ring for manipulation, which
the puppeteer wore as a crown on his head.
Some puppets have arms and hands together that have been replaced with sticks. Puppet puppets
are the largest, heaviest, and most vocal of all traditional Indian puppets. A puppet can be up to 4.5
feet long and weigh about ten kilograms. The Bommalattam Theater has wide openings divided into
four parts namely Vinayaka Puja, Komali, Amanattam, and Pusenkanattam.

33.Samvad Kaumudi was started by?

[A] Madhusudan Ray


[B] Raja Ram Mohan Roy
[C] Dadabhai Nauroji
[D] Devendranath Tagore

Hide Answer

Correct Answer: B [Raja Ram Mohan Roy]


Notes:
It was a Bengali weekly newspaper published in Kolkata in the first half of the 19th century by Ram
Mohan Roy. It was a well-known pro-Reformist book that campaigned for the abolition (standing) of
Sati Pratha.

34.Hindoo patriot was started by?

[A] Raja Ram Mohan Roy


[B] Dadabhai Nauroji
[C] Devendranath Tagore
[D] Madhusudan Ray

Hide Answer

Correct Answer: D [Madhusudan Ray]


Notes:
The Hindu Patriot weekly newspaper, first published on January 6, 1853, under the patronage of one
Madhusudhan Roy in association with the Girish Chandra Ghosh as Editor-in-Chief, changed
ownership in June 1855. Haran Chandra Mukherjee, the older brother of Harish Chandra Mukherjee
of Bawanipur became the first owner. The real buyer of the paper, however, was Harish Chandra
who had to keep himself safe from the Military Auditor General who worked under him.
The Hindu Patriot under Harish Chandra has played an important role in counteracting the pressure
of indigo cultivators, especially during the post-coup protests. Common editorial articles against such
violence in poor indigo raiyats have attracted public attention and sparked an international outcry
from a large part of the educated Indians.

35.Who wrote Kamasutra?

[A] Shankardev
[B] Vatsyayan
[C] Shudrak
[D] Shushruth

Hide Answer

Correct Answer: B [Vatsyayan]

Notes:
Kamasutra is an ancient Indian Sanskrit text on sensuality, sensuality, and emotional fulfillment in
life. Attributed to Vatsyayana, the Kama Sutra is neither exclusively nor primarily a sex manual on
sex positions, but rather the art of living well, the nature of love, finding life partners, and
maintaining one’s love life. and was written as a guide to others. Aspects relating to the pleasure-
oriented faculties of human life.

36.Kasavu is a soft, white handloom cotton fabric with gold thread borders that originated in which
South Indian state?
[A] Tamil Nadu
[B] Kerala
[C] Andhra Pradesh
[D] Karanataka

Hide Answer

Correct Answer: B [Kerala]

Notes:
Kasavu is a soft, white handloom cotton fabric with gold thread borders that originated in Kerala,
South Indian State. The Kasavu saree is famously worn by the Malayali community women on the
occasion of Onam.

37.In which of the following states is the Lippan Kam handicraft famous?

[A] Gujarat
[B] West Bengal
[C] Karnataka
[D] Jammu and Kashmir

Hide Answer

Correct Answer: A [Gujarat]


Notes:
‘Lipan Kaam’’ means mirror and clay work. It is a traditional craft of Kutch (Gujarat). It presents a
different style by using wild ass and camel dung as binding agents. Clay and clay are taken in equal
proportion, while it looks stunning with embroidered patterns and mirrors.

38.Which Veda focuses on the rites and mantras of different types of sacrifice?

[A] Rig Veda


[B] Yajur Veda
[C] Sama Veda
[D] Atharva Veda

Hide Answer

Correct Answer: B [Yajur Veda]

Notes:
The name ‘Yajus’ refers to ‘sacrifice’ and is centered on Vedic rituals and the various types of Yagya
mantras prevalent in Vedic times. There are two main sections in the Yajurveda (Samhitas): Shukla
(White / pure) and Krishna (black/dark). These codes are also known as:
Vajasanei Samhita and Taittiriya Samhita. Yajurveda acting as a guide for sages/priests is basically a
ritualistic Veda for those who perform sacrifices.

39.In which literary sources are the reference to Purushartha?

[A] Mahabharata
[B] Ramayana
[C] Bhagwat Gita
[D] Malavikiagnimitra

Hide Answer

Correct Answer: B [Ramayana]


Notes:
The most famous and revered text of the Ramayana was written by a sage Valmiki was one of the
earliest poets or poets. This logic is called Ramayana in Adikavya or poetry.
Although there has been a lot of talk about Ramayana dating, for the most part, BC Historians claim
that it was first compiled in 1500. In this epic, through the story of Rama who is portrayed as an ideal
man, Instructions on how to achieve the fourfold goals (masculine) of Mankind is given: Dharma,
Artha, Kama, moksha.

40.Radhakant Ray is a famous poet of which language?

[A] Kashmiri Literature


[B] Assamese literature
[C] Marathi literature
[D] Odia literature

Hide Answer

Correct Answer: D [Odia literature]


Notes:
Radhanath Ray (28 September 1848 – 17 April 1908) was an early modern Odia writer in Odia poetry
in the late nineteenth century. He was born into a zamindar family now in Baleshwar (Bengal
Presidency), Odisha and was honoured with the title of Kobibir in Oriya literature. In his early life, he
composed in both Oriya and Bengali but later switched to Oriya. He was born on September 28,
1848, in the village of Kedarpur in the Belashwar district of Odisha. He contributed poetry and
poetry to Oriya literature in the nineteenth century.

41.Baidehisha Bilasa, Koti Brahmand Sundari and Labanyabati are considered as milestones of Odia
literature, it is the creation of which of the following?

[A] Abhimanyu Samantasinhara


[B] Gopabandhu Das
[C] Fakir Mohan Senapati
[D] Upendra Bhanja

Hide Answer

Correct Answer: D [Upendra Bhanja]


Notes:
The works of Upendra Bhanja such as Baidehisha Bilasa, Koti Brahmand Sundari, and Labanyabati are
considered milestones of Odia literature.

42.Mirza Kalich Beg and Diwan Kaura Mal are litterateurs related to which language?

[A] Sindhi
[B] Gujarati
[C] Marathi
[D] Awadhi

Hide Answer

Correct Answer: A [Sindhi]

Notes:
Mirza Kalich Beg and Diwan Kaura Mal are litterateurs related to Sindhi language.
Sindh was one of the important centers of the Sufis, who established Khanqahs at various places.
Sufi singers popularized this language with their devotional music.

43.Ellora is an archaeological site located in Aurangabad, Maharashtra, India. They were built by the
rulers of which dynasty?

[A] Chalukya
[B] Rastrakutas
[C] Cholas
[D] Chandela

Hide Answer

Correct Answer: B [Rastrakutas]


Notes:
Ellora is an archaeological site located in Aurangabad, Maharashtra, India. These were built by the
rulers of the Rashtrakuta dynasty. Famous for its memorial caves, is a UNESCO World Heritage Site.

44.In which state is the monument to Pattadakal located?

[A] Tamil Nadu


[B] Kerala
[C] Karanataka
[D] Arunachal Pradesh

Hide Answer

Correct Answer: C [Karanataka]

Notes:
Pattadakallu is a town in the Indian state of Karnataka, which is famous for a group of monuments
containing the earliest uses of the Vesara style of Indian architecture. These temples were built in
the 8th century. There are temples of both Dravidian (South Indian) and Nagara (North Indian or
Aryan) styles. Pattadakallu is located at a distance of 22 km from Badami, the capital of the Chalukya
dynasty of South India. The kings of the Chalukya dynasty built many temples here in the seventh
and eighth centuries. Aihole is considered a school of architecture

45.Dhuandhar Falls is located in which district of Madhya Pradesh?

[A] Bhopal
[B] Ujjain
[C] Jabalpur
[D] Hosangabad

Hide Answer

Correct Answer: C [Jabalpur]


Notes:
Dhuandhar Falls is a waterfall located in Jabalpur district of Madhya Pradesh state, which falls from
50 feet above the Narmada stream. Whose water starts flying like white smoke. That is why it is
called ‘smoke’. Dhuandhar Falls was included as a World Heritage Site by UNESCO on 20 May 2021.

46.Rani ki Vav is the famous stepwell (stepped well) located in which district of Gujarat state of
India?
[A] Surat
[B] Ahmedabad
[C] Patan
[D] Dwarka

Hide Answer

Correct Answer: C [Patan]

Notes:
Rani ki Vav is a famous stepwell (step well) located in Patan in the state of Gujarat, India. This picture
is painted on a ₹100 note by RBI (Reserve Bank of India) in July 2018 and on 22 June 2014, it was
included in UNESCO World Heritage Site.

47.On which date India adopted its national flag?

[A] May 22, 1956


[B] July 22, 1947
[C] January 24, 1947
[D] January 26, 1950

Hide Answer

Correct Answer: B [July 22, 1947]

Notes:
The constituent Assembly adopted the national flag on 22 July 1947. The national flag was hoisted
for the first time by Surendranath Banerjee in 1906 at Calcutta.

48.What was the classical tradition of Gita Govinda known as?

[A] Bhairavi
[B] Ashtapadi
[C] Bani Vidya
[D] Dhrupadi

Hide Answer

Correct Answer: B [Ashtapadi]

Notes:
Jayadeva’s Gita Govinda from the 12th century was perhaps the earliest musical composition sung in
the classical tradition called Ashtapadi music.

49.How many basic notes are there in Indian Classical Music?

[A] 5
[B] 7
[C] 10
[D] 12

Hide Answer

Correct Answer: B [7]

Notes:
Indian classical music has seven basic notes with five interspersed half-notes, resulting in a 12-note
scale. Unlike the 12-note scale in Western music, the base frequency of the scale is not fixed, and
intertonal gaps (temperament) may also vary. The performance is set to a melodic pattern called a
raga characterized in part by specific ascent (aroha) and descent (avaroha) sequences, “king” (vadi)
and “queen” (samavadi) notes and characteristic phrases (pakad).

50.Who was known as the father of Carnatic Music?

[A] Pushkaran
[B] Purandara Dasa
[C] Mihira Bhoj
[D] Ganga Kishore

Hide Answer

Correct Answer: B [Purandara Dasa]

Notes:
By the 16th and 17th centuries, there was a clear demarcation between Carnatic and Hindustani
music; Carnatic music remained relatively unaffected by Persian and Arabic influences. Purandara
Dasa, who is known as the “father (Pitamaha) of Carnatic music”, formulated the system that is
commonly used for the teaching of Carnatic music

1.Which among the following Charter Acts ended the commercial monopoly of East India Company
in India?

[A] Charter Act of 1793


[B] Charter Act of 1813
[C] Charter Act of 1833
[D] Charter Act of 1853

Hide Answer

Correct Answer: B [Charter Act of 1813]

Notes:
Charter Acts of 1813 was an Act of the Parliament of the United Kingdom which renewed the charter
issued to the British East India Company, and continued the Company’s rule in India. However, the
Company’s commercial monopoly was ended, except for the tea trade and the trade with China.
Reflecting the growth of British power in India.

2.Which of the following act formally made the Governor General of Bengal as Governor General of
India?

[A] Regulating act 1773


[B] Pitts India act 1784
[C] Charter Act 1813
[D] Charter Act 1833

Hide Answer

Correct Answer: D [Charter Act 1833 ]

Notes:
The Charter Act of 1833 made the Governor General of Bengal the Governor General of British India
and all financial and administrative powers were centralized in the hands of Governor General-in-
Council.

3.For reforms in which among the following, Hartog Commission was established?

[A] Society
[B] Education
[C] Civil services
[D] Economic reforms

Hide Answer

Correct Answer: B [Education]

Notes:
The British Indian government appointed a committee under Sir Philip Joseph Hartog to inspect the
matter of the growth of education in India. In the year 1929, the committee submitted its report. It
mainly focused its attention towards mass education.

4.In context with the Wood’s despatch, consider the following statements:

1. Wood’s Despatch of 1854 laid the foundation of present system of education in India.

2. It helped the educational institutions founded private efforts to obtain grants from
government of India

3. It made a provision of moral and religious education as important part of education

Which among the above is / are correct statements?

[A] Only 1
[B] 1 & 2
[C] 2 & 3
[D] 1, 2 & 3

Hide Answer

Correct Answer: B [1 & 2]

Notes:
Wood’s Despatch of 1854 is called the Magnacarta of English Education in India. As per the act an
education department was to be set in every province. At least one government school was to be
opened in each and every district. Grant in Aid was also decided to provide in private schools.
Providing the training in the mother tongue of the Indian natives was decided. No provision of
religious education was entertained as per the recommendation.

5.In which year Hindu mahasabha was established?

[A] 1910
[B] 1915
[C] 1916
[D] 1918

Hide Answer

Correct Answer: B [1915]

Notes:
Hindu Mahasabha was founded in 1915 by Madan Mohan Malviya.It worked with Arya Samaj and
other Hindu organizations. It was directly link with Rashtriya Swam Sevak Sangh founded in 1925 at
Nagpur by K.B.Hegewar.The first All India Hindu Mahasabha Conference was organized at Hardwar
in 1915. The Sabha ecame more aggressive after 1929 and started propagating Hindu Rashtra which
was totally differ from Gandhiji’s Ram Rajya.

6.Which of the following movement was started to support Muslim resentment against ottoman
Empire Policy of British?

[A] Moplah Riots


[B] Khilafat Movement
[C] Reshmi Rumal Movement
[D] Ghadar Conspiracy

Hide Answer

Correct Answer: B [Khilafat Movement]

Notes:
The Khilafat Movement (1919-20 ) was essentially a movement to express Muslim support for the
Caliph of Turkey against the allied powers particularly Britain.

7.Consider the following incidences:

1. Foundation of Muslim league

2. Jallianwalla Bagh Massacre

3. Khilafat Movement

4. Civil Disobedience Movement

Arrange the above in Correct Chronological order:

[A] 1,2,3,4
[B] 1,3,2,4
[C] 2,1,3,4
[D] 2,4,1,3

Hide Answer

Correct Answer: A [1,2,3,4]

Notes:
Muslim league 1906,
Jallianwallla Bagh 13.4.1919,
Khilafat Movement 1919-1924,
Civil Disobedience 1930

8.James Augustus Hickey is also known as?


[A] Father of Indian media
[B] Father of Indian Press
[C] Father of Indian Television
[D] Pioneer of Photography in India

Hide Answer

Correct Answer: B [Father of Indian Press]


Notes:
James Augustus Hickey started the first Indian newspaper from Calcutta, the Calcutta General
Advertise or the Bengal Gazette in January, 1780.

9.On February 3, 1882, the British Government of India appointed an Indian Education Commission
1882, with a view to enquire into the working of the existing System of Public Instruction and further
extension of that system on popular basis. Who among the following was the chairman of this
commission?

[A] H P Jacob
[B] C Pearson
[C] G E Ward
[D] W.W. Hunter

Hide Answer

Correct Answer: D [W.W. Hunter]

Notes:
The Hunter Commission was appointed by Lord Ripon, the then viceroy of India with the
chairmanship of Sir William Hunter. Setting up of primary schools in rural areas, Establishment of
secondary schools by private parties, taking special care for the education of girls and women were
some of the important recommendation of the report of the commission.

10.Who among the following is best known for defending three Indian National Army soldiers
accused of treason during World War II?

[A] Asaf Al
[B] Bhulabhai Desai
[C] Subhash Chandra Bose
[D] C Rajgopalachari

Hide Answer

Correct Answer: B [Bhulabhai Desai]

Notes:
Bhulabhai Desai (1877 – 1946) was an acclaimed lawyer who defended three Indian National Army
soldiers accused of treason during World War II. He also attempted to negotiate a secret power-
sharing agreement with Liaquat Ali Khan of the Muslim League. (Bhulabhai-Laaqut Ali Talks)

11.The Universities of Calcutta, Bombay and Madras were established as the outcome of which
among the following?

[A] Hunter Commission Report


[B] Sir Charleswood’s dispatch
[C] Indian Universities Act
[D] Hartog Committee

Hartog Committee

Hide Answer

Correct Answer: B [Sir Charleswood’s dispatch]


Notes:
Wood’s Despatch of 1854 is called the Magnacarta of English Education in India. As per the act an
education department was to be set in every province. At least one government school was to be
opened in each and every district. Grant in Aid was also decided to provide in private schools.
Providing the training in the mother tongue of the Indian natives was decided. No provision of
religious education was entertained as per the recommendation. The Universities of Calcutta,
Bombay and Madras were also established.

12.Under which of the following INA comrades, Officers’ Training School for INA officers was opened
in 1943?

[A] Lt. Col. M.Z. Kiani.


[B] Habib ur Rahman
[C] Mohan Singh
[D] Lt. Col J.K. Bhonsle

Hide Answer

Correct Answer: B [Habib ur Rahman]


Notes:
Officers Training School for INA officers was opened under Habib ur Rahman. Azad School for the
civilian volunteers were set up to provide training to the recruits. For the first time in Asia a
women`s regiment, the Rani of Jhansi regiment was raised as a combat force, under Captain Laxmi
Swaminathan.

13.Who was the first and last Indian Governor-General of Independent India?
[A] Lord Wavell
[B] General Kodandera Madappa
[C] Chakravarti Rajgopalachari
[D] Lord Mountbaten

Hide Answer

Correct Answer: C [Chakravarti Rajgopalachari]

Notes:
Chakravarti Rajgopalachari (1948-50) was the first and last Indian Governor-General of Independent
India. He was the founder of the Swatantra Party and one of the first recipients of India’s highest
civilian award, the Bharat Ratna.

14.Who was the viceroy of India during Quit India Movement?

[A] Lord Linlithgow


[B] Lord Wavell
[C] Lord Irwin
[D] Lord Reading

Hide Answer

Correct Answer: A [Lord Linlithgow]


Notes:
Lord Linlithgow (1936 – 1944) was the viceroy of India during Quit India Movement. The Quit India
Movement (also known as the August Movement) was a civil disobedience movement launched by
Mahatma Gandhi on August 8,1942 in Mumbai for complete independence

15.Where did Indian society was founded by Anand Mohan Bose in 1872?

[A] Calcutta
[B] London
[C] Bombay
[D] Peshawar

Hide Answer

Correct Answer: B [London]

Notes:
In 1872, Anand Mohan Bose formed an Indian Society in London to foster the spirit of nationalism
among the Indian residents in Britain. Later, this organisation became an important platform for
Indians to voice their demands.

16.Who among the following personalities was the first President of the Progressive Writers Group?
[A] Munshi Prem Chand
[B] Bal Gangadhar Tilak
[C] Lala Lajpat Rai
[D] All of the above

Hide Answer

Correct Answer: A [Munshi Prem Chand]

Notes:
Munshi Prem Chand was elected as the first President of the Progressive Writers’ Association in
Lucknow, in 1936. The association was a progressive literary movement and sought to inspire people
through their writings advocating equality and attacking social injustice and backwardness.

17.What was / were the changes made into India’s governance by Government of India Act 1858?
1. Governor-General was to be called as the Viceroy
2. Office of Secretary of state for India was created
3. A highly centralised administrative structure was created
4. A federal court was established
Select the correct option from the codes given below:

[A] Only 1
[B] Only 1 & 2
[C] Only 1, 2 & 3
[D] 1, 2, 3 & 4

Hide Answer

Correct Answer: C [Only 1, 2 & 3]


Notes:
As per the Government of India Act 1858, the company rule was abolished and British rule started.
Board of Control and the Court of Directors were abolished. The post of the Secretary of State for
India was created with a total of 15 members in his council. Lord Stanley became the Secretary of
State for India. Governor-General was to be called as the Viceroy. As per Government of India Act of
1935, Federal Court was established in Delhi in the year 1937. Federal court came much later (GOI
Act 1935).

18.The most common identity of members of Commonwealth of Nations is they are former colonies
of British Empire. Which among the following countries are exception to this rule (i.e. they are not
former British colonies)?

1. Rwanda

2. Mozambique

3. Namibia

Select the correct option from the codes given below:


[A] Only 1 & 2
[B] Only 2 & 3
[C] Only 1
[D] 1, 2 & 3

Hide Answer

Correct Answer: A [ Only 1 & 2]

Notes:
British Empire, the Commonwealth’s only identity is that of a club of ex-colonies with Britain as its
president for life. (Rwanda and Mozambique, former German and French colonies, joining the
Commonwealth are odd exceptions to the rule.)

19.Which of the following is / are correct statements about the Charter Act of 1813?

1. It ended the monopoly of East India Company to trade in tea

2. It made the Governor General of Bengal the Governor General of British India

3. It also had a provision for Company to invest Rs. 1 Lakh every year on the education of
Indians.

Select the correct option from codes given below:

[A] 3 Only
[B] 2 and 3 Only
[C] 1,2 and 3
[D] None

Hide Answer

Correct Answer: A [ 3 Only]

Notes:
First statement is wrong because Charter act of 1833 ended the monopoly of EIC to trade in
Tea.Also, the second statement is wrong because it was Charter act of 1833 which made the
Governor general of bengal as Governor general of British India.Kindly,note the points of charter act
1813:

 Thus the Charter act of 1813 ended the monopoly of the East India Company in India,
however the company’s monopoly in trade with china and trade in tea was remained intact.

 The charter act of 1813, for the first time explicitly defined the constitutional position of the
British territories in India.

 This act also made provisions to grant permission to the persons who wished to go to India
for promoting moral and religious improvements. (Christian Missionaries).

 This act regulated the company’s territorial revenues and commercial profits. The company
debt was to be reduced and dividend was fixed @10.5% per annum.

 There was also a provision that Company should invest Rs. 1 Lakh every year on the
education of Indians.

 This act also empowered the local governments to impose taxes on the persons subject to
the jurisdiction of the Supreme Court.

20.Who was the first Indian to join the Viceroy’s executive council?

[A] G.K. Gokhale


[B] Mahatma Gandhi
[C] Satyendra Prasad Sinha
[D] Ambika Charan Majumdar

Hide Answer

Correct Answer: C [ Satyendra Prasad Sinha ]

Notes:
The Indian Councils Act 1909 provided for the first time the association of Indians with the executive
councils of Viceroys and Governors. Satyendra Prasad Sinha became the first Indian to join the
Viceroy’s executive council. He was appointed as a law member.

21.What was / were the changes made into India’s governance by Government of India Act 1858?
1. Governor-General was to be called as the Viceroy
2. Office of Secretary of state for India was created
3. A highly centralised administrative structure was created
4. A federal court was established
Select the correct option from the codes given below:

[A] Only 1
[B] Only 1 & 2
[C] Only 1, 2 & 3
[D] 1, 2, 3 & 4

Hide Answer

Correct Answer: C [Only 1, 2 & 3]


Notes:
As per the Government of India Act 1858, the company rule was abolished and British rule started.
Board of Control and the Court of Directors were abolished. The post of the Secretary of State for
India was created with a total of 15 members in his council. Lord Stanley became the Secretary of
State for India. Governor-General was to be called as the Viceroy. As per Government of India Act of
1935, Federal Court was established in Delhi in the year 1937. Federal court came much later (GOI
Act 1935).

22.With reference to the relations of British with the princely states, consider the following:

1. Policy of Ring Fence

2. Policy of Subordinate Co-operation and Union

3. Policy of Subordinate Isolation

Which among the following is the correct chronological order of the above?

[A] 1 2 3
[B] 1 3 2
[C] 2 1 3
[D] 3 2 1

Hide Answer

Correct Answer: B [ 1 3 2 ]

Notes:
There were 562 states with an area of 712, 508 sq. miles. Many states came into existence in the
later Mughal period. An examination of the relations between the British and the Indian states
suggests five stages :

 1740-1765. East India Company’s struggle for equality with Indian States.

 1765-1813. Policy of Ring Fence.

 18l3-1858. Policy of Subordinate Isolation.

 1858-1935. Policy of Subordinate Co-operation and Union.

 1935-1947. Policy of Equal Federation.

23.Consider the following statements about Tulsidas and Surdas:

1. While Tulsidas was a devotee of Lord Ram, Surdas was a devotee of Krishna

2. While Tulsidas is considered as Sagun bhakti saint, Surdas is considered as Nirguna bhakti
saint

Which of the above statements is/are correct?


[A] 1 Only
[B] 2 Only
[C] Both 1 & 2
[D] Neither 1 nor 2

Hide Answer
Correct Answer: A [ 1 Only]

Notes:
Second statement is incorrect because Surdas is considered a Saguna bhakti poet and so also known
as Sant Surdas, a name which literally means the "servant of melody". Nirguna saints (Kabir, Nanak,
Dadu Dayal, Namdev) were not limited to any particular sect or religion in their following and in their
ideas (thus were able to win followers among both Hindus and Muslims). But Sagun saints (Tulsidas,
Mira, Chaitanya, and Vallabhacharya) were tied to their particular sect.

24.When did the Portuguese lose Hooghly?

[A] 1531
[B] 1569
[C] 1632
[D] 1639

Hide Answer

Correct Answer: C [1632]


Notes:
Portuguese acquired Hooghly in 1537 and lost in 1562 during the reign of Mughal ruler Shahjahan.
25.Arrange Chronologically the settlement of the following European power in India
1. English
2. Portuguese
3. French
4. Dutch
Choose the correct option from the codes given below :

[A] 1, 2, 3, 4
[B] 4, 3, 1, 2
[C] 3, 1, 2, 4
[D] 2. 1, 4, 3

Hide Answer

Correct Answer: D [2. 1, 4, 3]

Notes:
The chronological arrangement of the settlement of the following European power in India-
Portuguese (1498), English (1600), Dutch (1602), Danes (1616), French (1664).

26.First Factory Act was passed during the viceroyship of who of the following?

[A] Lord Elgin


[B] Lord Ripon
[C] Lord Lytton
[D] Lord Lawrence

Hide Answer

Correct Answer: B [Lord Ripon]


Notes:
First Factory Act was passed in the year 1881 during the viceroyship of Lord Ripon. In the year 1885,
a Factory Commission was appointed. A Royal Labour Commission was appointed in the year 1891.
The main purpose of those commissions was to limit the working hours in the factory.

27.During the time of Lord Curzon, Dalai Lama of Tibet was influenced by who of the following?

[A] Yaroslav
[B] Boris
[C] Dorjieff
[D] Leonid

Hide Answer

Correct Answer: C [Dorjieff]

Notes:
During the time of Lord Curzon, Dalai Lama of Tibet was influenced by Dorjieff (He was a Russian).
That is why the English feared the alliance between Tibet and Russia.
28.Which among the following events took place during the viceroyship of Lord Hardinge II?
1. Delhi became the capital of India
2. Komagata Maru incident took place
3. Bihar and Orissa division was separated from the Bengal Presidency
4. Gopal Krishna Gokhale died
Choose the correct option from the codes given below:

[A] 1 & 4
[B] 1, 2 & 3
[C] 2, 3 & 4
[D] 1, 2, 3 & 4

Hide Answer

Correct Answer: D [1, 2, 3 & 4]

Notes:
Delhi became the capital of India in 1911. Komagata Maru incident took place in 1914. Bihar and
Orissa division was separated from the Bengal Presidency in the year 1912 (22nd March). Finally,
Odisha was separated from Bihar in the year 1936. Gopal Krishna Gokhale died in 1915.

29.The Raksha Bandhan ceremony was first celebrated on which date?

[A] 16th October 1906


[B] 16th October 1911
[C] 16th October 1905
[D] 16th July 1905

Hide Answer

Correct Answer: C [16th October 1905]

Notes:
On the day of the commencement of the Partition of Bengal (16th October 1905), Rabindranath
Tagore proposed to celebrate the Raksha Bandhan ceremony to strengthen the unity among all
religious groups.

30.Who was the head of the commission which was appointed by Congress to investigate the
Jallianwala Bagh Massacre?

[A] C.R. Das


[B] Motilal Nehru
[C] Ambika Charan Majumdar
[D] B.V Patel

Hide Answer

Correct Answer: A [C.R. Das]

Notes:
Congress appointed a commission under the leadership of C.R Das to investigate the Jallianwala
Bagh Massacre. Other members of the commission were Gandhiji, Motilal Nehru, Jayakar, Abbas
Tyabji.

31.Who of the following was the longest-serving Viceroy of India?

[A] Lord Minto II


[B] Lord Wavell
[C] Lord Linlithgow
[D] Lord Wellingdon

Hide Answer

Correct Answer: C [Lord Linlithgow]

Notes:
Lord Linlithgow served as the viceroy of India from 1936 to 1943. His seven years tenure made him
the longest-serving Viceroy of India.

32.Simon Commission reached India in which year?


[A] 1927
[B] 1928
[C] 1929
[D] 1931

Hide Answer

Correct Answer: B [1928]

Notes:
In the year 1928 (3rd February) Simon Commission reached India (Bombay). Lord Irwin was the
viceroy at that time. Indians raised their voice with the slogan “Simon Go Back”. on 16th February
Lala Lajpat Roy proposed the Boycott of the Commission.

33.When did Gandhiji move to South Africa?

[A] 1988
[B] 1890
[C] 1893
[D] 1905

Hide Answer

Correct Answer: C [ 1893 ]

Notes:
In the year 1893, Gandhiji went to South Africa in connection with a professional work to represent
an Indian merchant in a case as a lawyer. In the year 1915, he returned to India.

34.Which among the following about Lord Mayo’s resolution of 1870 are correct?
1. It was the first step which bifurcated central and provincial finances
2. Provincial Governments were empowered to administer certain services
3. It did not attempt to rectify existing imparity
4. It had never focused on the actual needs of the provinces
Choose the correct option from the codes given below :

[A] 1 & 2
[B] 1, 3 & 4
[C] 2, 3 & 4
[D] 1, 2, 3 & 4

Hide Answer

Correct Answer: D [1, 2, 3 & 4]

Notes:
Lord Mayo’s resolution of 1870 aimed at strengthening of Economy. It was the first step that
bifurcated central and provincial finances. As per the resolution, provincial Governments were
empowered to administer certain services. It did not attempt to rectify existing imparity. It had
never focused on the actual needs of the provinces.

35.Who among the following sacrificed her life to protect the rights of the peasants?

[A] Veera Gunnamma

[B] Rani Gaidinliu


[C] Madam Cama
[D] Aruna Asaf Ali

Hide Answer

Correct Answer: A [Veera Gunnamma


]

Notes:
Veera Gunnamma sacrificed her life to protect the rights of the peasants in Andhra Pradesh in the
year 1940.

36.The objective of which of the following organization was “to resuscitate and reinvigorate the
Congress”?
[A] Radical Democratic Party
[B] Hindustan Labour Kishan Party
[C] Congress Socialist Party
[D] All India Trade Union Congress

Hide Answer

Correct Answer: C [Congress Socialist Party]

Notes:
Acharya Narendra Deva said the objective of Congress Socialist Party was “to resuscitate and
reinvigorate the Congress”. In the year 1934, the Congress Socialist Party was founded.

37.Which of the following statements regarding Permanent Settlement is/are correct?


1. The system of Permanent Settlement was introduced in parts of the Madras and Bombay
presidencies.
2. It created a new class of landlords with hereditary rights on land.
3. The landlords created under Permanent Settlement could never be removed under any
circumstance.
Select the correct option from the codes given below:

[A] Only 1
[B] Only 2
[C] Only 2 & 3
[D] 1, 2 & 3

Hide Answer

Correct Answer: B [Only 2]

Notes:
The system of Permanent Settlement was introduced in the provinces of Bengal, Bihar and Odisha.
Under this system the landlords lost their proprietary rights over the lands if they couldn’t submit
the fixed revenue to Britishers on time.

38.Which of the following statements are correct?


1. Rani Laxmi Bai was the leader of the Indian Mutiny in Jhansi.
2. Begum Hazrat Mahal was the leader of the Indian Mutiny in Lucknow.
3. Azimullah Khan was the leader of the Indian Mutiny in Kanpur.
4. Moulavi Ahmadullah was the leader of the Indian Mutiny in Farrukhabad.
Select the correct option from the codes given below:
[A] 1, 2 & 3 only
[B] 2 & 3 only
[C] 1, 2 & 4 only
[D] 1, 2, 3 & 4

Hide Answer

Correct Answer: A [1, 2 & 3 only]

Notes:
1. Rani Laxmi Bai was the leader of the Indian Mutiny in Jhansi.
2. Begum Hazrat Mahal was the leader of the Indian Mutiny in Lucknow.
3. Azimullah Khan was the leader of the Indian Mutiny in Kanpur.
4. Moulavi Ahmadullah was the leader of the Indian Mutiny in Faizabad.

39.Which of the following statements is/are correct regarding the Permanent Settlement?
1.?The Permanent Settlement was introduced in parts of the Madras and Bombay Presidencies.
?2.?The Permanent Settlement created a new class of landlords with hereditary rights on land.
?3.?The landlords created by the Permanent Settlement could never be removed under any
circumstance.
Select the correct option from the codes given below:

[A] Only 1
[B] Only 2
[C] 2 and 3
[D] 1, 2 and 3

Hide Answer

Correct Answer: B [Only 2]

Notes:
As per the Permanent Settlement system, the Zamindars who formerly collected revenues were
“recognized” as Landlords and the ownership of the Land was made hereditary. This means that now
onwards there would be no auctioning. The son of Zamindar would be a Zamindar.

40.Which of the following was the first recorded Muslim revolutionary who was hanged for India’s
independence?

[A] Mohammad Ali


[B] Shaukat Ali
[C] Ashfaque-ullah-Khan
[D] Azizuddin

Hide Answer

Correct Answer: C [Ashfaque-ullah-Khan]

Notes:
Ashfaque-ullah Khan the first recorded Muslim revolutionary who was hanged for India’s
independence. Ashfaqullah Khan was detained in the Faizabad jail and a case was filed against him.
His brother Riyasatullah Khan was his legal counsel in the case.

41.Who of the following Generals was impeached in the British Parliament?

[A] Warren Hastings


[B] Lord Hastings
[C] Lord Clive
[D] None of the above

Hide Answer

Correct Answer: A [Warren Hastings]

Notes:
Warren Hastings was the first Governor of the Presidency of Fort William, the head of the Supreme
Council of Bengal, and thereby the first de facto Governor-General of India from 1772 to 1785. ?In
1787, Warren Hastings was impeached in the Parliament by Edmund Burke.

42.Which of the following statements are correct?


1. Warren Hastings was the first Governor-General who established a regular police force in India on
the British pattern.
2. A Supreme Court was established at Calcutta by the Regulating Act, 1773.
3. The Indian Penal Code came into effect in the year 1860.
Select the correct answer from the codes given below:

[A] 1 and 2
[B] 2 and 3
[C] 1 and 3
[D] 1, 2 and 3

Hide Answer

Correct Answer: B [2 and 3]


Notes:
1. Lord Cornwallis was the first Governor-General who established a regular police force in India on
the British pattern.
2. A Supreme Court was established at Calcutta by the Regulating Act, 1773.
3. The Indian Penal Code came into effect in the year 1860.

43.Which of the following is the correct chronological order of the following events:
1.Abolition of dual government in Bengal
2.Treaty of Allahabad
3.Battle of Plassey
4.Battle of Wandiwash
Select the correct option from the codes given below:

[A] 1, 2, 3, 4
[B] 2, 3, 4, 1
[C] 3, 4, 2, 1
[D] 4, 1, 3, 2

Hide Answer

Correct Answer: C [3, 4, 2, 1]


Notes:
1. The Battle of Plassey occurred in 1757.
2. The Battle of Wandiwash occurred in 1760.
3. The treaty of Allahabad held in 1765.
4. The abolition of the dual Government in Bengal took place in 1772.

44.Which of the following statements are correct?


1. Rani Laxmi Bai was the leader of the Indian Mutiny in Jhansi.
2. Begum Hazrat Mahal was the leader of the Indian Mutiny in Lucknow.
3. Azimullah Khan was the leader of the Indian Mutiny in Kanpur.
4. Moulavi Ahmadullah was the leader of the Indian Mutiny in Farrukhabad.
Select the correct option from the codes given below:

[A] 1, 2 & 3 only


[B] 2 & 3 only
[C] 1, 2 & 4 only
[D] 1, 2, 3 & 4

Hide Answer

Correct Answer: A [1, 2 & 3 only]

Notes:
1. Rani Laxmi Bai was the leader of the Indian Mutiny in Jhansi.
2. Begum Hazrat Mahal was the leader of the Indian Mutiny in Lucknow.
3. Azimullah Khan was the leader of the Indian Mutiny in Kanpur.
4. Moulavi Ahmadullah was the leader of the Indian Mutiny in Faizabad.

45.Which of the following were the intentions of British Indian rule behind the partition of Bengal?
1. To create a majority of Bengal against non-Bengalis in the parent Bengal.
2. To keep Bengali, Oriya, and Hindi speaking people together in the parent Bengal.
3. To create a majority of Muslims in the East Bengal.
Select the correct option from the codes given below:

[A] 1 and 2 only


[B] 2 and 3 only
[C] 1 and 3 only
[D] 1, 2 and 3

Hide Answer

Correct Answer: B [2 and 3 only]

Notes:
The intention of British Indian rule behind the partition of Bengal was :
1. To create a majority of Bengal against non-Bengalis in the parent Bengal.
2. To keep Bengali, Oriya, and Hindi speaking people together in the parent Bengal.
3. To create a majority of Muslims in the East Bengal.

46.Eastern Bengal and Assam were created in which of the following years?

[A] 1905
[B] 1907
[C] 1911
[D] 1913

Hide Answer

Correct Answer: A [1905]

Notes:
Eastern Bengal and Assam were created in the year 1905 upon the partition of Bengal, together with
the former province of Assam. It was re-merged with Bengal in 1912.
47.The integration of most of the princely states in the Union of India took place in which of the
following years?

[A] 1948
[B] 1947
[C] 1950
[D] 1961

Hide Answer

Correct Answer: B [1947]

Notes:
The integration of most of the princely states in the Union of India took place in 1947. All princely
states merged in the Union of India except Junagarh, Jammu and Kashmir and Hyderabad.

48.Which among the following was the correct order of the four-step integration?

[A] Reorganisation-Merger-Democratisation-Centralisation and constitutionalisation


[B] Merger-Democratisation-Centralisation and constitutionalisation-Reorganisation
[C] Democratisation-Centralisation and constitutionalisation-Reorganisation-Merger
[D] Reorganisation-Merger-Centralisation and constitutionalisation-Democratisation

Hide Answer

Correct Answer: B [Merger-Democratisation-Centralisation and constitutionalisation-


Reorganisation]

Notes:
The correct order of the four-step integration was Merger-Democratisation-Centralisation and
constitutionalisation-Reorganisation.

49.Who among the following was shot dead while hoisting the National Flag at Gohapon police
station?

[A] Sarojini Naidu


[B] Kanaklata Barua
[C] Maniben Patel
[D] Matangini Hazra

Hide Answer

Correct Answer: B [Kanaklata Barua]

Notes:
Kanaklata Baruah was one of the most cherished martyrs of freedom movement. She was also called
as Birbala. She had taken part in the Quit India Movement and was a leader of the procession aimed
at hoisting the national flag at Gohapon police station. She was shot dead while hoisting the
National Flag.

50.Parvati Agrawal was the daughter of which among the following Indian freedom fighter?

[A] Bal Gangadhar Tilak


[B] Gopal Hari Deshmukh
[C] Rasbehari Bose
[D] Lala Lajpat Rai

Hide Answer

Correct Answer: D [Lala Lajpat Rai]

Notes:
Parvati Devi was daughter of Lala Lajpat Rai who is know for actively taking part in the Satyagraha
Committee. Radha Devi Agrawal was the wife of Lala Lajpat Rai. He had two sons, Amrit Rai Agrawal
and Pyarelal Agrawal, and a daughter, Parvati Agrawal

1.Which among the following Bodhisattva holds thunderbolt?

[A] Vajrapani
[B] Manjushri
[C] Maitreya
[D] Amitabha

Hide Answer

Correct Answer: A [ Vajrapani ]

Notes:
The Bodhisattva, Vajrapani, holds a thunderbolt like the deity Indra of Hinduism and fights against
evil and sin. Avlokitesvara or Padmapani is the lotus bearer; Majushri holds a book describing 10
paramitas ( spiritual perfections); Maitreya is the future Buddha; Ksitigarbha is the bodhisattva of
hell-beings and guardian of purgatories; while Amitabha is Buddha of heaven and the Buddha of
Immeasurable Life and Light.

2.The grandson of Ashoka, who accepted Jainism was __?

[A] Samprati
[B] Kunala
[C] Dasharatha
[D] Salisuka

Hide Answer

Correct Answer: A [ Samprati ]

Notes:
Emperor Samprati was the grandson of Emperor Ashoka, who reigned from 224–274 BCE. And
accepted Jainism.

3.In which rock edict Ashoka mentioned “Every Human is my child…”?


[A] Major Rock Edict V
[B] Major Rock Edict IV
[C] Major Rock Edict VI
[D] Major Rock Edict VII

Hide Answer
Correct Answer: A [ Major Rock Edict V ]

Notes:
In the major Rock Edict V, Ashoka mentioned “Every Human is my child”. He had concerns about the
policy towards slaves. This rock edict also referes to the appointment of Dhamma-mahamatta for
the first time in the twelfth year of his reign.

4.The caves and rock-cut temples at Ellora are __:

[A] Buddhist
[B] Buddhist and Jain
[C] Hindu and Jain
[D] Hindu, Buddhist and Jain

Hide Answer

Correct Answer: D [ Hindu, Buddhist and Jain ]

Notes:
Ellora, built by Rashtrakutas is located 29 km North-West of Aurangabad in Maharashtra. It is one of
the World Heritage Sites. It is well known for its monumental caves viz. 12 Buddhist (caves 1–12), 17
Hindu Caves (caves 13–29) and 5 Jain Caves (caves 30–34). These caves were built in proximity and
demonstrate the religious harmony prevalent during this period of Indian history.

5.The Kalinga war was fought in which year of Ashoka reign?

[A] 6
[B] 7
[C] 8
[D] 9

Hide Answer

Correct Answer: C [ 8 ]

Notes:
The Magadha Emperor, Ashoka invaded Kalinga in the eight year of his reign in 261 B.C. Nearly one
lakh soldiers lost their lives in that war and one and half lakh soldiers were captured. The 13th rock
edict of Ashoka throws light on this war.

6.The place Dharanikota in South India is related to which ancient dynasty?

[A] Chola
[B] Satavahana
[C] Pandya
[D] Chera

Hide Answer

Correct Answer: B [ Satavahana ]

Notes:
Dharanikota is a town near Amaravati in the Guntur district of Andhra Pradesh in India, It is the site
of the ancient Dhanyakataka which was the capital of the Satavahana kingdom which ruled in the
Deccan around the 1st to 3rd centuries A.D.
7.Who was the chairman of Third Tamil Sangam?

[A] Nakkirar
[B] Agastaya
[C] Tiruttakkadevar
[D] Chintamani

Hide Answer

Correct Answer: A [ Nakkirar ]

Notes:
Nakkirar was the chairman of 3rd Tamil Sangam held at North Madurai.

8.Which king of Ancient India has been given the title” Indian Nepoleon” by VA Smith?

[A] Samudragupta
[B] Kumaragupta
[C] Chandragupta Vikramaditya
[D] Skandagupta

Hide Answer

Correct Answer: A [Samudragupta]

Notes:
Samudragupta believed in the policy of war and conquest and because of his bravery and
generalship he is known as the “Napoleon of India” by historian Vincent. A. Smith.

9.The Excavated Buddhist Site, Lalitgiri , which was notified as Centrally Protected in the year 1937,
is located in which among the following states of India?

[A] Andhra Pradesh


[B] Odisha
[C] West Bengal
[D] Karnataka

Hide Answer

Correct Answer: B [Odisha]

Notes:
Lalitgiri is situated in Cuttack district of Odisha which is at a distance of around 100 kms from
Bhubaneswar. The Buddhist relics found during excavation was notified as centrally protected in the
year 1937.

10.Which century Gautama Buddha was born in?


[A] 4th century BC
[B] 5th century BC
[C] 6th century BC
[D] 7th century BC

Hide Answer

Correct Answer: C [6th century BC]


Notes:
Gautam Siddhartha was the name of Buddha and he was the founder of Buddhism also known as
Gautam Buddha. Gautam Buddha was born in 563 BC in Lumbini (now in Nepal) in Sakya Kshatriya
clan of Kapilvastu. He died in 483 BC.

11.For the first time in Indian History, the description of the State Administration was found for
which among the following periods?

[A] Gupta Period


[B] Maurya Period
[C] Shunga Period
[D] Satavahana period

Hide Answer

Correct Answer: B [Maurya Period]

Notes:
For the first time in Indian History, the description of the State Administration was found for
Mauryan period in the Arthashashtra of Kautilya. The Mauryan Empire was divided into four
provinces with the imperial capital at Pataliputra.

12.The Royal seal of Guptas bore the emblem of _________?

[A] Hasti (Elephant)


[B] Mayura ( Peacock)
[C] Garuda (Eagle)
[D] Tiger

Hide Answer

Correct Answer: C [Garuda (Eagle)]


Notes:
The royal emblem used by the Gupta period was Garuda. It is the mythic eagle Garuda, the dynastic
symbol of the Guptas.

13.Aihole is called the cradle of Indian temple architecture. Which among the following dynasty is
related to the construction of magnificent temples in Aihole?

[A] Chalukya
[B] Chola
[C] Chera
[D] None of the above

Hide Answer

Correct Answer: A [Chalukya]


Notes:
The Aihole temples were built in the late 7th century by the dynasty of the Chalukyas. It is the largest
of a group of over 120 temples at Aihole. The architecture of the temple is predominantly Dravida
with Nagara style also is used in certain areas.
14.which among the following sentence is incorrect about Rajadhiraja Chola I?

[A] He was a born fighter son of Rajendra Chola I


[B] He died in the battle of Koppan fighting with Chalukyas
[C] He assumed the title of Parakesari
[D] All are correct

Hide Answer

Correct Answer: C [He assumed the title of Parakesari]

Notes:
Parakesari and Yuddhamalla were the titles of Rajendra 1
Rajadhiraja Chola assumed the title of Rajakesari

15.Vajasneya belongs to the following Veda?

[A] Rigveda
[B] Sama Veda
[C] Yajurveda
[D] Atharvaveda

Hide Answer

Correct Answer: C [Yajurveda]


Notes:
Vajasneya samhita belongs to Shukla YajurVeda. The name Vajasaneyi is derived from Vajasaneya,
the patronymic of Yajnavalkya, and the founder of the Vajasaneyi branch.

16.As per the ancient Indian Texts, the land between which among the following two rivers has been
called “Brahamavarta”?

[A] Saraswati river and Drsadvati River


[B] Saraswati river and Saryu River
[C] Sutudri (Satluj) River and Vipas (Beas) River
[D] Ganga River and Yamuna River

Hide Answer

Correct Answer: A [Saraswati river and Drsadvati River]

Notes:
The Hindu religious text Manusmriti describes Brahmavarta as the region between the rivers
Saraswati and Drishadwati in India.

17.Consider the following caves of historical importance in India?

1. Ajanta Caves

2. Elephanta Caves

3. Barabar Caves

Which among the above is / are Deccan Trap caves?


[A] Only 1
[B] Only 1 & 2
[C] Only 2 & 3
[D] 1, 2 & 3

Hide Answer

Correct Answer: B [Only 1 & 2]

Notes:
Some of the best known Deccan Trap caves are close to Mumbai (Bombay), including Ajanta
(perhaps the oldest one dating back to 200 B.C.), Mandapesvara Caves, Kanheri Caves, Jogeshwari
Caves, Mahakali Caves, and Elephanta Caves.

18.Which of the following terms with respect to Chola administration denote some kind of taxes?

1. Idaivari

2. Alugalsarakku

3. Angadipattam

4. Uppa-yam

Select the correct option from the codes given below:

[A] 1 2 3
[B] 2 3 4
[C] 1 3 4
[D] 1 2 3 4

Hide Answer

Correct Answer: D [ 1 2 3 4 ]

Notes:

1. Idaivari – It was a tax on weights

2. Alugalsarakku – It was a tax or fine on rotten medicines / drugs

3. Angadipattam – It was a tax on Bazaars

4. Uppa-yam – It was a salt tax

19.Which among the following can be inferred on the basis of the observations of Fa-Hien, who
visited India during the Gupta era?

1. Practice of non-violence was common

2. Absence of Barter trade

3. Rigid caste system had developed

Select the correct option from the codes given below:

[A] Only 1 & 2


[B] Only 2 & 3
[C] Only 1 & 3
[D] 1, 2 & 3

Hide Answer

Correct Answer: C [Only 1 & 3]

Notes:
Fa Hien visited during the reign of the Gupta ruler, Chandragupta-II. He informs the presence of the
Hinayana doctrine in the Gandhara, Bannu, Kanauj, and Kausambi region while of the Mahayana
doctrine in Khotan. In Afghanistan, Punjab, Mathura, and Pataliputra both the Hinayana and
Mahayana doctrines were popular according to him. He named the region south of Mathura as
Madhyadesha, and provided a much idealized picture of ordinary life of the people in this region. He
mentions that people practiced non-violence. The animal killing, liquor drinking, eating of onion or
garlic was absent. He further mentions that there was an absence of corporal punishment and
criminals were generally made to pay fines for their wrongs. The royal officials were paid in cash (it
was later changed to land grants) . The chandalas lived outside the cities and before entering into
the city they were to inform the people so that people can avoid their touch and even shadow.

20.Consider the following observations with respect to the Harappan settlements:

1. The majority of Harappan settlements were located near water sources

2. Harappans made sinking wells and building reservoirs, where local surface water supplies
were inadequate

Which among the above statements is / are correct?

[A] Only 1
[B] Only 2
[C] Both 1 & 2
[D] Neither 1 nor 2

Hide Answer

Correct Answer: C [Both 1 & 2]

Notes:
Access to water is an important consideration in settlement sites the world over. The majority of
Harappan settlements were located near water sources such as streams and aquifers, but the
Harappans were competent hydraulic engineers, sinking wells (as at Mohenjo-daro) and building
reservoirs (as at Dholavira) where local surface water supplies were inadequate. They dealt with the
problems of river flooding, annually variable in its location and force and a potential deterrent to
settlement in some areas, by constructing many settlements with massive flood defences in the
form of platforms and walls. 21.With reference to the Aranyakas, which among the following
statements is / are correct?

1. They are parts of Brahamans

2. They focus on rites, rituals and sacrifices

3. Their content is generally esoteric / philosophical

Select the correct option from the codes given below:


[A] Only 1 & 2
[B] Only 1 & 3
[C] 1, 2 & 3
[D] Only 3

Hide Answer

Correct Answer: B [ Only 1 & 3 ]

Notes:
The Aranyakas were written in Forests and are concluding parts of the Brahmans. Aranyakas don’t
lay much emphasis on rites, ritual and sacrifices but have philosophy and mysticism. So they have
moral science and philosophy. It also provides the details of the rishis who lived in jungles. The end
portions of many Brahmanas have an esoteric content, called the ‘Aranyakas’

22.Consider the following observations about Sabha and Samiti in Rigvedic times:

1. While Sabha was a selected body, Samiti was assembly of larger group of people

2. While Sabha performed most judicial as well as political functions, Samiti did not do these
functions

Which of the above is / are correct?

[A] Only 1
[B] Only 2
[C] Both 1 & 2
[D] Neither 1 nor 2

Hide Answer

Correct Answer: A [ Only 1]

Notes:
Two assemblies called ‘sabha’ and ‘samiti’ formed an essential feature of the government. The term
‘sabha’ is often mentioned in the Rigveda, and denotes both “the people in conclave” and the “hall”
which was the venue of their meeting. Since, however, the ‘ sabha ‘ was used for the game of dice, it
is clear that even nonpolitical business could be transacted at the hall or by the people who
constituted the ‘ sabha’. That it was a gathering of the elect i.e., of Brahmanas and the rich patrons,
when it was concerned for administrative purposes, is clear from the term ‘ Sabheya’, “worthy of the
assembly”, as applied to a Brahmana. The ‘samiti’ in the sense of an “Assembly” of the Vedic tribe is
mentioned in the Rigveda. According to Ludwig, the’ samiti’ was a more comprehensive conference
including not only all the common people (visah) but also Brahmanas and rich patrons (maghavan).
Although it is difficult to distinguish between a ‘sabha’ and a ‘samiti’, it appears that the ‘ samiti ‘
was an august assembly of a larger group of the people for the discharge of tribal (i.e. political)
business and was presided over by the king, while the ‘ sabha’ , a more select body, was less popular
and political in character than the ‘samiti’ . Although the functions and powers of ‘ sabha’ and ‘
samiti’ cannot be exactly defined, numerous passages referring to them clearly indicate that both
these Assemblies exercised considerable authority and must have acted as healthy checks on the
power of the king. Source: Administrative System in India: Vedic Age to 1947 By U. B. Singh

23.Which of the following Rock Edicts of Asoka talks about taxation?


[A] Bhabru Inscription
[B] Nigalisagar Pillar Inscription
[C] Rummindei Pillar Inscription
[D] Barabar Cave Inscription

Hide Answer

Correct Answer: C [Rummindei Pillar Inscription]

Notes:
Rummindei Pillar Inscription says that The Beloved of the Gods, the king Piyadassi, when he had
been consecrated twenty years, came in person find referenced the place where Buddha Sakyamuni
was born. He caused a stone enclosure to be made and a stone pillar to be erected. As the Lord was
born here in the village of Lumbini, he has exempted it from tax, and fixed its contribution [i.e. Of
grain] at one-eigth.

24.Consider the following pairs:

1. Burzahom culture – Jammu And Kashmir

2. Ganeshwar culture – Madhya Pradesh

3. Jorwe culture – Maharashtra

Which of the above is/are correct?


[A] 1 & 3 Only
[B] 3 Only
[C] 2 & 3 Only
[D] 1, 2 & 3

Hide Answer

Correct Answer: A [ 1 & 3 Only]

Notes:
Ganeshwar is located near the copper mines of the Sikar-Jhunjhunu area of the Khetri copper belt in
Rajasthan. Jorwe culture is a Deccan Chalcolithic culture, located in Ahmednagar district of
Maharashtra. Burzahom was the first Neolithic site to be discovered in Kashmir. Burzahom was the
first Neolithic site to be discovered in Kashmir. It is located on a “karewa” between the banks of the
Dal Lake and the Zabarvan hills, about 5 km from the famous Mughal garden of Shalimar.

25.Who among the following is known as father of Indian archaeology?

[A] Colonel Mackenzie


[B] Alexander Cunningham
[C] Robert Seawell
[D] John Marshal

Hide Answer

Correct Answer: B [Alexander Cunningham ]


Notes:
Alexander Cunningham is the first Director-General of the Archaeological Survey of India (ASI). He is
often called as the father of Indian archaeology.

26.Which of the following is concerned with Vedic sacrifices that require three or more fires?

[A] Shrautasutra
[B] Grihyasutra
[C] Dharmasutra
[D] None of the above

Hide Answer

Correct Answer: A [Shrautasutra]

Notes:
‘Vedanga’ means the ‘limbs of the Vedas’. The Vedanga consist of six sutras. Shrautasutra which is a
part of Kaplasutra is concerned with Vedic sacrifices that require three or more fires.

27.Which event of Gautam Buddha’s life is depicted by the wheel and deer in Indian art?

[A] Great departure


[B] Enlightenment
[C] First preaching
[D] Nirvana

Hide Answer

Correct Answer: C [First preaching]

Notes:
The first preaching event of Gautam buddha’s life is represented by the wheel and deer in Indian art.
Mahatama Buddha gave the first preaching or sermon at Mrigadava Sarnath.

28.What was a market town called in 6th century B.C?

[A] Nagarka
[B] Mahanagara
[C] Nigama
[D] Rajdhani

Hide Answer

Correct Answer: C [Nigama]

Notes:
During 600 – 300 BCE, a market town which was at an intermediate level between the Grama and
Nagara was called as Nigama. These market places were often crowded with merchants.

29.What was the capital city called during the 6th century BCE?

[A] Mahanagara
[B] Rajdhani
[C] Nigama
[D] Pura
Hide Answer

Correct Answer: B [Rajdhani]

Notes:
The beginning of the 6th century BCE was marked with an increase in the settlements and
corresponding growth of population in the Ganga valley. The name of the capital city was Rajdhani.

30.According to Divyavadana, Bindusara wanted which of the following to succeed him?

[A] Susima
[B] Ashoka
[C] Tissa
[D] None of the above

Hide Answer

Correct Answer: A [Susima]

Notes:
There was a four year succession conflict after the death of Bindusara in 273 BCE. According to
Divyavadana, Bindusara
wanted his elder son Susima to succeed him. But Bindusara’s ministers wanted Ashoka to succeed.

31.Which of the following is the term used for traders and artisans organised in corporate
associations in the Mauryan period?

[A] Shulka
[B] Ahitakas
[C] Shrenis
[D] None of the above

Hide Answer

Correct Answer: C [Shrenis]

Notes:
The traders and artisans during the Mauryan period were organised in corporate associations called
shrenis or guilds or puga to protect their rights. These guilds were headed by Jesthaka.

32.Indo-Greeks invaded India in which of the following century?

[A] 1st century BCE


[B] 2nd century BCE
[C] 3rd century BCE
[D] 4th century BCE

Hide Answer

Correct Answer: A [1st century BCE]

Notes:
After the death of Ashoka, the successors of Ashoka were too weak to hold ground. Therefore a
series of invasions on the
north-western border of India started. In the beginning of the 2nd century BCE, the Indo-Greeks
were the first to invade the Indian subcontinent.

33.Which of the following had imprisoned Harsha’s sister Rajyashri and killed her husband
Grahavarman?

[A] Devagupta
[B] Shashanka
[C] Dhruvasena II
[D] Pulkesin II

Hide Answer

Correct Answer: B [Shashanka]

Notes:
Harsha succeeded his brother Rajya Vardhana. He undertook a campaign against the ruler of Malwa,
Devagupta and Shashanka, who was the ruler of Gauda. Shashanka had imprisoned Harsha’s sister
Rajyashri and killed her husband Grahavarman.

34.Which of the following was called as the Commander-in-Chief of Harsha?

[A] Avanti
[B] Simhananda
[C] Bhanu
[D] Kuntala

Hide Answer

Correct Answer: B [Simhananda]

Notes:
Harsha Vardhana ruled from 606 A.D. to 647 A.D. He maintained the administrative set-up of his
empire on the model of previous great Hindu rulers. The Commander-in-Chief during the Harsha’s
administration was called as Simhananda.

35.Which of the following kings succeded Mahipala II?

[A] Surapala
[B] Ramapala
[C] Kumarapala
[D] Vigrahapala III

Hide Answer

Correct Answer: A [Surapala]

Notes:
Mahipala II was killed in a revolt led by an officer of the state, Divya. Surapala succeded the throne
after Mahipala II who ruled for about a couple of years and then he was succeeded by his younger
brother Ramapala.

36.Which of the following was the founder of Kadamba dynasty?


[A] Vidhyashakti
[B] Mayursharman
[C] Kakusthavarma
[D] None of the above

Hide Answer

Correct Answer: B [Mayursharman]

Notes:
The Kadamba dynasty was founded by Mayurasharman. Mayurasharman defeated the Pallavas with
the help of forest tribes. The Pallavas formally presented Mayurasharman with royal insignia to give
recognition to his authority.

37.Which of the following statements are correct regarding the Varna System during the early
medieval period?
1. Varnas or social classes refused to discharge the functions assigned to them.
2. The lower orders arrogated to themselves the status and functions of the higher orders.
3. Their was refusal to pay taxes and render labour services.
Select the correct option from the codes given below:

[A] Only 1
[B] Only 1 & 2
[C] Only 2 & 3
[D] 1, 2 & 3

Hide Answer

Correct Answer: D [1, 2 & 3]

Notes:
Varnas or social classes during the early medieval period refused to discharge the functions assigned
to them. The lower orders assumed the status and functions of the higher orders. Their was refusal
to pay taxes and render labour services.

38.Which of the following are correctly matched regarding import in India from 650 A.D. to 1000
A.D.?
Items of import Region
1. Horses Western and Central Asia
2. Wine Kapisa
3. Dates Basrsa
Select the correct option from the codes given below:

[A] Only 1
[B] Only 2
[C] Only 1 & 3
[D] 1, 2 & 3

Hide Answer

Correct Answer: D [1, 2 & 3]


Notes:
Between 650 A.D. to 1000 A.D. the best breed of horses came to India from the Western and Central
Asia. Wine was imported from Kapisa and dates were imported from Basra.

39.Which of the following was the author of Tattvasuddhi?

[A] Jayanta
[B] Nathamuni
[C] Haribhadra
[D] Udayana

Hide Answer

Correct Answer: D [Udayana]

Notes:
Tattvasuddhi was written by Udayana in 984 AD. He was Hindu logician of tenth century. He
attempted to reconcile the views held by two major schools of logic called as Nyaya and Vaisheshika.

40.Which of the following kings Succeeded King Krishna I?

[A] Dantidurga
[B] Dhruva
[C] Amoghavarsha I
[D] Govind III

Hide Answer

Correct Answer: B [Dhruva]

Notes:
King Dhruva of the Rashtrakuta empire ruled the kingdom from 780 A.D. to 793 A.D. Dhruva was one
of the most notable rulers of the Rashtrakuta Empire. During his reign, the Rashtrakuta kingdom
expanded into an empire that encompassed all of the territory between the Kaveri River and Central
India
41.Which of the following statements are correct with respect to Rashtrakutas?
1. Rashtrakuta rule lasted in the Deccan till the end of the 10th century.
2. They were not tolerant in their religious views.
3. The great apabhramsha poet Swayambhu and his son probably lived at the Rashtrakuta court.
Select the correct option from the codes given below:

[A] Only 1
[B] Only 1 & 2
[C] Only 1 & 3
[D] 1, 2 & 3

Hide Answer

Correct Answer: C [Only 1 & 3]

Notes:
The Rashtrakuta rule lasted in the Deccan till the end of the 10th century. They were tolerant in their
religious views. The great apabhramsha poet Swayambhu and his son is believed to have lived at the
Rashtrakuta court.
42.Which of the following Chalukyan King set up a pillar of victory on the shores of the Southern
Ocean?

[A] Vikramaditya I
[B] Vinayaditya I
[C] Vijayaditya
[D] Vikramaditya II

Hide Answer

Correct Answer: D [Vikramaditya II]

Notes:
Vikramaditya II is believed to have destroyed the Cholas, Keralas, Pandyas, Kalabhras and other
kings. He also set up a pillar of victory on the shores of the Southern Ocean.

43.Which of the following Chola king allied with the Pallavas and defeated Pandyas and as a
recompense received some
territories in the Tanjore area?
[A] Vijayalaya
[B] Aditya I
[C] Rajaraja I
[D] Rajadhiraja

Hide Answer

Correct Answer: B [Aditya I]

Notes:
Aditya I who ruled from c.871 to 907 CE allied with the Pallavas and defeated Pandyas and as a
recompense received some
territories in the Tanjore area, at the battle of Shripurambiyam.

44.The famous dancing girl found in the Mohenjodaro was made up of which material?

[A] Bronze
[B] Red Limestone
[C] Steatite
[D] Terracotta

Hide Answer

Correct Answer: A [Bronze]

Notes:
The famous ‘Dancing girl’ found in Mohenjodaro is an artifact that is some 4,500 years old. The 10.8
cm long statue of the dancing girl was found in 1926 from a house in Mohenjodaro is made of
bronze. Steatite (Soap Stone) artifacts have been found at nearly every excavated Harappan period
site and were the primary element used to make seals. Terracotta were used to make figurines.

45.Which one of the followings is Indus Valley site in Pakistan?


[A] Harappa
[B] Kalibangan
[C] Lothal
[D] Alamgirpur

Hide Answer

Correct Answer: A [Harappa]

Notes:
Harappa is an archaeological site situated in Punjab, Pakistan. The site takes its name from a modern
village located near the former course of the Ravi River. Harappa contains the ruins of a Bronze Age
fortified city, which was part of the Cemetery culture and the Indus Valley Civilization, centered in
Sindh and the Punjab.

46.In which country Shortugai, an Indus Valley Civilization site is located?

[A] Pakistan
[B] Afghanistan
[C] Tibet
[D] India

Hide Answer

Correct Answer: B [Afghanistan]

Notes:
Shortugai, in Darqad District of northern Afghanistan, was a trading colony of the Indus Valley
Civilization established around 2000 BC on the Oxus river near the lapis lazuli mines. It is considered
to be the northernmost settlement of the Indus Valley Civilization. Traces of canals have been found
at this site. It is possible that ancient canals silted up long ago. It is also likely that water drawn from
wells was used for irrigation.

47.Which one of the following were the silver coins from Gupta period?

[A] Rupaka
[B] Dinar
[C] Kakini
[D] Nishka

Hide Answer

Correct Answer: A [Rupaka]

Notes:
The silver coins issued during the Gupta period were called ‘Rupaka.’ These coins were based on
Sakas of Ujjaini weighing 32-36 grains. However, according to Fa-Hsien, cowries were the common
means of exchange.

48.Who compiled the tales of “The Panchatantra”?

[A] Tulsidas
[B] Vishnu Sharma
[C] Vedavyasa
[D] Valmiki

Hide Answer
Correct Answer: B [Vishnu Sharma]

Notes:
The Panchatantra is attributed to Vishnu Sharma. It an ancient Indian collection of interrelated
animal fables in verse and prose arranged within a frame story. The original Sanskrit work, some
scholars believe was composed around the 3rd century BC.

49.Who founded the famous Vikramashila University?

[A] Dharmpala
[B] Gopala
[C] Devapala I
[D] Devapala II

Hide Answer

Correct Answer: A [Dharmpala]

Notes:
Vikramashila University was established by King Dharmapala of the Pala dynasty in the 8th century
A.D. Located at Bhagalpur in modern day Bihar, it was one of the two most important centres of
Buddhist learning in India during the Pala empire, along with Nalanda. It developed into the
intellectual center for Tantric Buddhism.

50.Which ruler deafeated Harsha Vardhan in 618 CE?

[A] Pulakeshin II
[B] Chandragupta I
[C] Pushyamitra
[D] Alexander

Hide Answer

Correct Answer: A [Pulakeshin II]

Notes:
According to the Aihole inscription, Chalukyan ruler Pulakeshin II defeated Harsha Vardhan on the
banks of Narmada in 618-619 A.D. Pulakeshin entered into a treaty with Harsha, with the Narmada
River designated as the border between them.

1.The irrigation tax was charged on the farmers for the first time by which among the following
Sultans?

[A] Alauddin Khilji


[B] Ghiyasuddin Tughlaq
[C] Mohammad Tughlaq
[D] Firoz Tughlaq

Hide Answer

Correct Answer: D [Firoz Tughlaq]


Notes:
Firoz Shah Tughlaq was the cousin brother of Muhammad Bin Tughlaq, and became the ruler of
Delhi on 23rd march 1351 A.D. He imposed only four taxes sanctioned by Islamic viz., kharaj (land
tax), khams (1/5 of the looted property during wars), Jizya (relig

2.The Mansabdari system was borrowed from ?

[A] Afghanistan
[B] Turkey
[C] Mongolia
[D] Persia

Hide Answer

Correct Answer: C [Mongolia]

Notes:
Mansabdar implies the generic term for the military-kind grading of all royal functionaries of the
Mughal Empire. The Mansabdari system introduced by Akbar was borrowed from the system
followed in Mongolia.

3.Which among the following was the real name of Mughal Emperor Muhammad Shah ?
[A] Khujasta Akhtar
[B] Shahriyar Shah Bahadur
[C] Nadir Shah
[D] Raushan Akhtar

Hide Answer

Correct Answer: D [Raushan Akhtar]

Notes:
Raushan Akhtar was the real name of Muhammad Shah.

4.The ambassador of Emperor James I, who reached in the court of Jahangir in 1615 was __:
[A] Sir James Hay
[B] Sir Thomas Roe
[C] Sir Thomas Howard
[D] Sir John Digby

Hide Answer

Correct Answer: B [Sir Thomas Roe]

Notes:
As an ambassador of Emperor James l, Sir Thomas Roe reached in the court of Mughal Emperor
Jehandri at Agra in 1615. Jehangir presented him the Mansab of 400.

5.Baburnama was written in which language?

[A] Para-Mongolic
[B] Chagatai Turkic
[C] Hijazi Arabic
[D] Bahrani Arabic

Hide Answer

Correct Answer: B [Chagatai Turkic]

Notes:
Babur has written his biography i.e. Baburnama which is also known as Tuzk-e Babri. Babur and
Jahangir are the only two emperors of Mughal Empire who wrote their own biographies. Baburnama
is also known to be the first true autobiography in the Islamic li

6.The infamous Third Battle of Panipat was fought between which among the following belligerents?

[A] British and Marathas


[B] Mughals and Marathas
[C] Afghans and British
[D] Marathas and Afghans

Hide Answer

Correct Answer: D [Marathas and Afghans]

Notes:
Third Battle of Panipat was fought in 1761. This was the largest war of the 18th century. The
Marathas were led by Sadashivrao Bhau. The battle followed a two months seige and skirmishes.
The Marathas were badly defeated in the war suffering a life of around 60-70 thousand warriors.
They lost their ablest commanders. Balaji Baji Rao could not absorb this shock and died soon
afterwards.

7.The silver coins of which among the following kings carried portraits and bilingual legends, which
were inspired by the Kshatrapa types?

[A] Satavahanas
[B] Kushanas
[C] Guptas
[D] Mauryas

Hide Answer

Correct Answer: A [Satavahanas ]

Notes:
Satavahana kings are noted for issuing bilingual coins featuring Middle Indo-Aryan language on one
side, and Tamil language on the other side.

8.In whose times the Mughal Empire reached its territorial climax ?
[A] Akbar
[B] Shah Jahan
[C] Aurangzeb
[D] Bahadur Shah I

Hide Answer
Correct Answer: C [Aurangzeb]

Notes:
The Mughal empire reached territorial climax during the reign of Aurangzeb. He ruled from 1658 to
1707. Aurangzeb policies lead to the dissolution of the empire.

9.Maritime power was a distinct character of which of the following dynasties?

[A] Chalukyas
[B] Cholas
[C] Pandyas
[D] Rasthrakutas

Hide Answer

Correct Answer: B [Cholas]

Notes:
Maritime power was a distinct character of the Cholas. The cholas rules from Sri Lanka to Indonesia.
They has one of the greatest navies of the time.

10.Which of the following rulers had turned the Moti Masjid of Lahore built by Mughal emperor
Shah Jahan into a gemstone repository?

[A] Jahangir
[B] Shah Jahan
[C] Furrukhsiyar
[D] Maharaja Ranjit Singh

Hide Answer

Correct Answer: D [Maharaja Ranjit Singh]

11.Mahzarnama was declared by ____?

[A] Babur
[B] Humayun
[C] Akbar
[D] Shahjahan

Hide Answer

Correct Answer: C [Akbar]

Notes:
In 1579, Mahzar Nama was declared by which the emperor declared that if there were conflictions in
the views of the debaters, he was entitled to choose any of the interpretations.
With the Mahzar Nama, Akbar pounced upon the dominance of the intolerant orthodox and allowed
free development of a genuine religious spirit. Mahzar Nama was actually an idea of the father of
Abul Fazal and Faizi , set that the authority of the King was higher than that of a Mujtahid (doctor of
the faith) and if there is a variance, the emperor’s decision should be binding on the Muslims of
India. With this edict, Akbar’s judgment was set above every legal and religious authority, so it was
the promulgation of the doctrine of Imperial infallibility.
12.Manyakheta (modern Malkhed) on the banks of Kagina River in Gulbarga district, Karnataka state
was the capital of ______?

[A] Satvahanas
[B] Rastrakutas
[C] Hoysalas
[D] Kakatiyas

Hide Answer

Correct Answer: B [Rastrakutas]

Notes:
Rashtrakuta was a royal dynasty ruling large parts of the Indian subcontinent between the sixth and
10th century. They ruled Karnataka and the capital is Manyakheta (modern Malkhed).

13.The word coromandal cost has been derived from the kingdom of which of the following?

[A] Cheras
[B] Cholas
[C] Chalukyas
[D] Pandyas

Hide Answer

Correct Answer: B [Cholas]

Notes:
The name coromandal occurs in the forms Cholamandalam or Solamandalam on the great temple
inscription of Tanjore. So Coromandel takes its name from the ancient dynasty of Tamil rulers known
as the Cholas.

14.Which among the following is not correct ?

[A] The capital of pandyas was Madurai


[B] The capital of Cheras was Vanchi
[C] Capital of the Videha Kingdom – Mithila
[D] All are correct

Hide Answer

Correct Answer: D [All are correct]

Notes:
The Pandyas ruled the south India and capital was Madurai. The cheras also ruled the south India
with their capital Vanchi. The capital of Videha Kingdom is Mithila.

15.Consider the following statements with respect to Iqtadari system:

1. It guaranteed administrative, military and judicial powers to officers

2. Iqtadar did not enjoy hereditary powers

3. Iqta was the largest unit of administration under the system

Which of the above statements is/are correct?


[A] 3 Only
[B] 1 and 2 Only
[C] 2 and 3 Only
[D] 1,2 and 3

Hide Answer

Correct Answer: C [2 and 3 Only]

Notes:
It guaranteed administrative, military and judicial powers to officers
This is an incorrect statement because the system did not guarantee judicial powers. It only
guaranteed administrative and military powers.
Iqtadar did not enjoy hereditary powers
This is a correct statement.
Iqta was the largest unit of administration under the system
This is also a correct statement.

16.Who among the following defeated the Rajputs at Chanderi in 1528?


[A] Humayun
[B] Akbar
[C] Babur
[D] Jahangir

Hide Answer

Correct Answer: C [Babur]

Notes:
The Battle of Chanderi took place in the aftermath of the Battle of Khanwa in which the Mughal
Emperor Babur had defeated a confederacy of Rajputs and Afghans which was headed by Rana
Sanga of Mewar. Babur himself expressed surprise that the upper fort had fallen within an hour of
the final assault.

17.Ramanuja, the Vaishnava Saint, was born at which of the following places?

[A] Srirangam
[B] Srivilliputtur
[C] Udipi
[D] None of the above

Hide Answer

Correct Answer: D [None of the above]

Notes:
Ramanuja was born to mother Kanthimathi and father Asuri Kesava Somayaji, in Sriperumbudur,
near modern Chennai, Tamil Nadu. He is believed to have been born in the month of Chaitra under
the star Tiruvadhirai.

18.Who wrote the Shahjan-Nama that covers Shah Jahan’s entire reign?
[A] Inayat Khan
[B] Muhammad Sadiq Khan
[C] Abul Hamid Lahori
[D] Muhammad Waris

Hide Answer

Correct Answer: B [Muhammad Sadiq Khan]

Notes:
Other than Inayat Khan, another Shahjahan-Nama was written by Muhammad Sadiq Khan, who was
a waqia-navis at the Mughal court. Sadiq was a selfless freelance writer who undertook the work just
for the love of it and recorded the events of Shah Jahan’s reign from the death of Jahangir to the
accession of Aurangzeb in an intelligent and impartial manner.

19.In which battle, the confederacy led by Hindu Shahi army of Anandapala was defeated by
Mahmud of Ghazni?

[A] Battle of Janjua


[B] Battle of Peshawar
[C] Battle of Chach
[D] Battle of Takeshar

Hide Answer

Correct Answer: C [Battle of Chach]

Notes:
Battle of Chach was fought in 1008 AD between Mahmud of Ghazni and a confederacy of Hindu
Kingdoms led by Hindu Shahi army of Anandapala. Forces of Anandpala were joined by Ajmer,
Kalingar, Kanauj etc. Anandpala lost the battle mainly because the army was not well organized and
his elephant took flight in battle.

20.Which of the following was not a regional Hindu Kingdom in power at the time of attacks of
Mohammad Ghori on India?

[A] Tomars of Delhi


[B] Gahadavlas of Kannauj
[C] Solankis of Gujarat
[D] Karkota Empire of Kashmir

Hide Answer

Correct Answer: D [Karkota Empire of Kashmir]

Notes:
The principal Hindu powers of Northern India about this time (end of 12th century) included
Tomaras of Delhi, Gahadavlas, also known as Rathors of Kanauj, Chauhan of Ajmer, Solankis /
Parmaras/ Bagheals of Gujarat and Malwa; and Palas and Senas of Bihar and Bengal. The Karkota
dynasty was active between 625 – 885 AD.

21.The Slave dynasty was also called as?


[A] Khilji dynasty
[B] Mameluk dynasty
[C] Lodi dynasty
[D] Ghurid dynasty

Hide Answer

Correct Answer: B [Mameluk dynasty]

Notes:
The Slave dynasty was also called the Mameluk dynasty. ‘Mameluk’ was an Arabic word which
means ‘owned’. It was used to distinguish the important Turkish slaves which were meant for
military service from the lower slaves (domestic labour or artisans).

22.Jalaluddin Mangbarani sought shelter from Mongols, from which of the following?

[A] Iltutmish
[B] Ruknuddin Firuz Shah
[C] Bahram Shah
[D] Alauddin Masud Shah

Hide Answer

Correct Answer: A [Iltutmish]

Notes:
Jalaluddin Mangbarani who was the son of the Shah of Khwarizm, while escaping from the Mongols,
sought shelter from Iltutmish but Iltutmish refused to grant asylum to him. By doing so
Iltutmish prevented Mongol attack and saved his sultanate from the wrath of Chengiz Khan.

23.Which was first Delhi sultan to assume title of Zil-i-Ilahi?

[A] Balban
[B] Ruknuddin Firuz Shah
[C] Bahram Shah
[D] Nasiruddin Mahmud

Hide Answer

Correct Answer: A [Balban]

Notes:
According to Balban who ruled from 1266 CE to 1286 CE, the Sultan was God’s shadow on earth also
called Zil-i-Ilahi and the recipient of divine grace also called as Nibyabat-i-Khudai.

24.Which of the following department was called as Diwan-i-unzarat during the reign of Balban?

[A] Finance Department


[B] Military Department
[C] Foreign Department
[D] None of the above

Hide Answer

Correct Answer: A [Finance Department]


Notes:
Balban who ruled the Delhi sultanate from 1266 CE to 1286 CE. Balban was undoubtedly one of the
main architects of the Delhi Sultanate. He separated the Finance Department also called as Diwan-i-
unzarat from the Diwan-i-Arz (Military Department).

25.During the reign of which of the following Devagiri was invaded by Alauddin Khalji?

[A] Qutbuddin Mubarak Shah


[B] Nasiruddin Khusrau Shah
[C] Jalauddin Khalji
[D] None of the above

Hide Answer

Correct Answer: C [Jalauddin Khalji]

Notes:
During the reign of Jalauddin Khalji in c.1294 CE, Devagiri was invaded by Alauddin Khalji also called
Ali Gurshap during the reception there in July c. 1296 CE.
26.Which of the following was appointed as Amir-i-Tuzuk (Master of Ceremonies) by Jalauddin
Khalji?
[A] Alauddin Khalji
[B] Mubarak Shah
[C] Khusrau Shah
[D] None of the above

Hide Answer

Correct Answer: A [Alauddin Khalji]

Notes:
Alauddin Khalji was the nephew and son-in-law of Jalaluddin Khalji. He had helped his uncle in his
struggle for power. He was appointed as Amir-i-Tuzuk or the Master of Ceremonies.

27.Safarnama Rehla was written by which of the following authors?

[A] Fakhruddin
[B] Ibn Batuta
[C] Amir Khusrau
[D] Minhaj-us-Siraj

Hide Answer

Correct Answer: B [Ibn Batuta]

Notes:
Ibn Battuta was a Muslim Berber Moroccan scholar, and explorer who widely traveled the medieval
world. During the reign of Mohammad Bin Tughlaq, he came to India. He was the author of
Safarnama Rehla.

28.The Atal Devi Masjid in Jaunpur was constructed during the reign of which of the following kings?
[A] Malik Sarwar
[B] Mubarak Shah
[C] Ibrahim Shah
[D] Khizr khan

Hide Answer

Correct Answer: C [Ibrahim Shah]

Notes:
Atala Masjid was originally a temple of goddess Atala devi built by Kannauj King Vijaychandra. It was
partially destroyed and converted to a mosque in 14th century by Ibrahim Shah, the Sharqi dynasty
ruler of Jaunpur sultanate. The so called new design was called Sharqi architecture.

29.In the Sultanate period the governors of the provinces were known as?

[A] Muqtis
[B] Shiqdar
[C] Amil
[D] Patwari

Hide Answer

Correct Answer: A [Muqtis]

Notes:
But the governors of the provinces in the Sultanate period were known as the muqtis or walis. They
task of muqtis was to maintain law and order and collect the land revenue.

30.The Portuguese travellers Domingo Paes and Barbosa came to India during his reign of which of
the following?

[A] Deva Raya I


[B] Deva Raya II
[C] Krishna Deva Raya
[D] Achyuta Deva Raya

Hide Answer

Correct Answer: C [Krishna Deva Raya]

Notes:
King Albuquerque sent his ambassadors to the court of Krishna Deva Raya as he maintained friendly
relations with the portuguese. The Portuguese travellers Domingo Paes and Barbosa came to India
during reign of Krishna Deva Raya only.

31.Which of the following kings transferred his capital from Delhi to Devagiri?
[A] Firoz Shah Tughlaq
[B] Abu Bakr Shah
[C] Mohammad Bin Tughlaq
[D] Ghiyaddin Tughlaq Shah II

Hide Answer
Correct Answer: C [Mohammad Bin Tughlaq]

Notes:
Mohammad Bin Tughlaq also called as Jauna Khan ruled the Delhi sultanate from 1325 CE to 1351
CE. According to Ziauddin Barni, Mohammad Bin Tughlaq Transferred his capital from Delhi to
Devagiri (Daulatabad).

32.Which of the following was the author of ‘Tarikh-i-Firoz Shah’ and ‘Futwa-i-Jahandari’?

[A] Barani
[B] Khwajja Abdul Malik Islami
[C] Baduani
[D] None of the above

Hide Answer

Correct Answer: A [Barani]

Notes:
Firoz Shah Tughlaq patronised scholars like Barani. Barani was a famous historian who wrote Tarikh-
i-Firoz Shah and Futwa-i-Jahandari. Firoz Shah Tughlaq himself wrote the Futuhat-e-firozshahi.

33.The First Battle of Panipat was held in which of the following years?

[A] 1520
[B] 1526
[C] 1530
[D] 1535

Hide Answer

Correct Answer: B [1526]

Notes:
The first Battle of Panipat was held in 1526 A.D. Ibrahim Lodhi was defeated and killed. With him,
the Sultanate period ended. Babur was invited by Daulat Khan Lodi to invade India.

34.Who founded the independent Jaunpur Sultanate?

[A] Ibrahim Shah


[B] Mahmud Shah
[C] Khwajah-i-Jahan Malik Sarwar
[D] Hussain Shah

Hide Answer

Correct Answer: C [Khwajah-i-Jahan Malik Sarwar]


Notes:
The Jaunpur Sultanate was an independent sultanate that was in power from 1394 to 1479. It was
founded by Khwajah-i-Jahan Malik Sarwar of Sharqi dynasty. Sarwar was a former wazir of Sultan
Nasiruddin Muhammad Shah IV Tughluq. He took the advantage of disintegration of Tughlaq dynasty
and became an independent ruler of Jaunpur Sultanate extending his authority over awadh and
large part of Ganga-Yamuna Doab.
35.Which god was Medhpateshwar?

[A] Lord Shiva


[B] Lord Vishnu
[C] Lord Agni
[D] Lord Brahma

Hide Answer

Correct Answer: A [Lord Shiva]

Notes:
The rulers of Mewar believed themselves to be the custodians of the Hindu civilization epitomized in
the temple of their lord Eklingaji which is a manifestation of Lord Shiva also called Medhpateshwar
(Lord of Medhpath).

36.Which of the following constructed a palace in Delhi called the Kushk-i-Lal?

[A] Qutubuddin Aibak


[B] Iltutmish
[C] Aram Shah
[D] Balban

Hide Answer

Correct Answer: D [Balban]

Notes:
Balban of the Slave dynasty constructed a palace in Delhi called the Kushk-i-Lal before he became
king. He got constructed Qila-i-Marzghan near this palace after he became king.

37.The famous temple of Tirupati was developed during the period of which of the following ruler of
Vijayanagara?

[A] Deva Raya I


[B] Deva Raya II
[C] Krishna Deva Raya
[D] Achyuta Deva Raya

Hide Answer

Correct Answer: C [Krishna Deva Raya ]

Notes:
The famous temple of Tirupati was greatly developed during the period of Krishna Deva Raya as the
deity there was his titular deity. He also built some fine stone temples and got repaired a number of
temples.

38.Tiruvengalanatha temple was constructed during the reign of which of the following kings?

[A] Vira Narasimha Raya


[B] Krishna Deva Raya
[C] Achyuta Deva Raya
[D] Sada Siva Raya
Hide Answer

Correct Answer: C [Achyuta Deva Raya]

Notes:
The Tiruvengalanatha temple was built at Vijayanagara during the reign of the king Achyuta Deva
Raya. Tiruvengalanatha temple is now popularly known as the Achyutaraya temple on the name of
this king.

39.Chand Bibi was the daughter of which of the following?

[A] Hussain Nizam Shah I


[B] Burhan I
[C] Burhan II
[D] Murtuza I

Hide Answer

Correct Answer: A [Hussain Nizam Shah I]

Notes:
Chand Bibi was the daughter of Hussain Nizam Shah I. Chand Bibi was married to Ali Adil Shah I of
Bijapur. She took over the reigns of power in her parental kingdom of Ahmednagar to stand in
opposition against the Mughals.

40.Akbar appointed which of the following as the governor of Multan in 1509?

[A] Khan-i-Khanan
[B] Raja Todar Mal
[C] Raja Man Singh
[D] Mirza Aziz Koka

Hide Answer

Correct Answer: A [Khan-i-Khanan]

Notes:
Akbar appointed Khan-i-Khanan as the governor of Multan in 1509 A.D. Akbar gave him the
responsibility of subduing Bilochis, a tribe in the Sindh region, and to conquer the whole territory

41.The followers of the Krishna cult founded the Radha Ballabhi sect under which of the following?

[A] Tulsidas
[B] Hari Vamsa
[C] Sur Das
[D] Chaitanya

Hide Answer

Correct Answer: B [Hari Vamsa]

Notes:
In c.1585 CE, the followers of the Krishna cult founded the Radha Ballabhi. The sect was founded
under Hari Vamsa. Vallabacharya, a popular bhakti saint popularised the Krishna bhakti cult in the
early 16th century.
42.Which of the following was the most prominent king of the Sabiri branch of Chisti order?

[A] Miranji Shams al Ushshaq


[B] Abdul Quddus Gangohi
[C] Shaikh Piyara
[D] Sayyid Yadullah

Hide Answer

Correct Answer: B [Abdul Quddus Gangohi]

Notes:
One of the most well-known saints of Sabiri branch of Chisti order was Abdul Quddus Gangohi, who
along with other Sufis revived the old glory of the Chishtis in the North.

43.Which of the following taught yoga to his disciple Sulaiman Mandavi?

[A] Miranji Shams al Ushshaq


[B] Abdul Quddus Gangohi
[C] Shaikh Piyara
[D] Sayyid Yadullah

Hide Answer

Correct Answer: B [Abdul Quddus Gangohi]


Notes:
Abdul Quddus Gangohi was the most prominent saint of the Sabiri branch of the Chisti order. He was
deeply interested in yoga and is reported to have taught it to his disciple, Sulaiman Mandavi, in
return for lessons in Quranic recitation.

44.In which year, Akbar built Buland Darwaza at Fatehpur Sikri in order to commemorate the victory
over Gujarat?

[A] 1572
[B] 1575
[C] 1579
[D] 1582

Hide Answer

Correct Answer: B [1575]

Notes:
Akbar defeated the Gujarat ruler Muzaffar Shah without any serious resistance and in order to
commemorate the victory over Gujarat, Akbar built the Buland Darwaza at Fatehpur Sikri in 1575.

45.Which of the following jagirs were given on certain conditions under the Jagirdari system?

[A] Tankha Jagirs


[B] Mashrut Jagirs
[C] Watan Jagirs
[D] Altamgha Jagirs

Hide Answer
Correct Answer: B [Mashrut Jagirs]

Notes:
Mashrut Jagirs under the Jagirdari system of Mughals were the lands which were given on certain
conditions. Tankha Jagirs were those which were given in lieu of salaries and they were transferable
every three to four years.

46.Tansen accepted Islam at the hand of which of the following great Sufi Saint?

[A] Salim Chishti


[B] Sheikh Muinuddin Chisti
[C] Qutbuddin Bakhtiar Kaki
[D] Muhammad Ghaus

Hide Answer

Correct Answer: D [Muhammad Ghaus]

Notes:
Tansen was a Hindu of Gwalior and also a great musician. He was one of the courtiers who were
collectively known as Akbar’s Navratnas or the nine jewels. He also served as a court musician to
King Ramachandra. He accepted Islam at the hand of great Sufi mystic saint Muhammad Ghaus of
Gwalior.

47.Jahangir ruled during which of the following periods?

[A] 1605–1627 A.D.


[B] 1615–1630 A.D.
[C] 1620–1637 A.D.
[D] 1635–1642 A.D.

Hide Answer

Correct Answer: A [1605–1627 A.D.]

Notes:
Jahangir or Salim ruled from 1605 to 1627 A.D. Jahangir revolted against Akbar after Akbar’s death,
he was crowned at Agra in 1605 A.D. He assumed the title of Nur-ud-din Muhammad Jahangir which
means conqueror of the world.

48.British visited Machlipatnam during the reign of which of the following?

[A] Akbar
[B] Jahangir
[C] Aurangzeb
[D] Shahjahan

Hide Answer

Correct Answer: B [Jahangir]

Notes:
The British visited Machlipatnam in south India during the reign of Mughal emperor Jahangir. Both
Captain Hawkins (c.1608–11 CE) and Thomas Roe (c.1615–19 CE) visited his court.
49.During the reign of which of the following, Nikusiyar, the grandson of Aurangzeb, revolted and set
up himself as Emperor at Agra?

[A] Farrukh Siyar


[B] Rafi-us-Darajat
[C] Rafi-us-Daula
[D] Mohd. Shah Rangeela

Hide Answer

Correct Answer: B [Rafi-us-Darajat]

Notes:
During the reign Rafi-us-Darajat, Nikusiyar who was the grandson of Aurangzeb, revolted and set up
himself as Emperor at Agra with the help of Mitrasen (a Nagar Brahmin). Rafi-us-Darajat ruled in
c.1719 CE.

50.Which of the following Peshwa helped Shahu in his rise to power?

[A] Shyampant Kulkarni Ranzekar


[B] Moropant Trimbak Pingle
[C] Balaji Vishwanath
[D] Sonopant Dabir

Hide Answer

Correct Answer: C [Balaji Vishwanath]

Notes:
The period of the rule of the king Shahu was also marked by the ascendancy of a lineage of
Chitpavan Brahmin ministers, who held the title of Peshwa. The first truly prominent figure of this
line was Balaji Vishwanath, who had helped Shahu in his rise to power.

You might also like